You are on page 1of 40

Mathematical re~O<mmg ie a part of rcrbal I.

Letter Sene"
rea>omng. For UGC-1\'IlT mathemancal 2 Numh>·Sel'!e>
reaMn:ng we hove to re~<i onl;c 3.Ldter and :o-:mnheJ' rcliwrl Serres
1. Serie• Comrlonon Generally I-CC ask que,IH·ns b""~d onlr on
2. CoOing und Decoding

om
number series a:>J ]pttcr sene"-
~- Cbwfioatlon (Odd ~l~n Out)
4. Analogical P..elalionohip ILETTER SERmsl
The WOt'(( verbal is Oelln<'tl as ponmnin~ to In the kt!<Jt' "~"'~"' rome lelt~ro arc given 1 hat
words rath~1· than thmgs. Verbal T~mohin~ full<>W a pnrlLCular -'~quonce or m>:b·. Y cu h' ,.,
tests use wnrrl, l~tl~r and number.<, and rcquil'e '" d~t~ct tho Jl"tco?rn from (.he pvon l~tt('ro ~nd
log1cal reao"ru:og ~nrl a reclSunah'~ ktwwledg~ of lind the missing lott~r or th~ nn1 lett<-r to

.c
the Enghsh language_ lr. lS aL>o neccoaary to he cnntmue the. pltern.
familiar wit.h simele mampubtinno II'Jlh Ilini-S to Solt e th P Letter SPdes
figurco, lihe addrtion, 8Ubtractwlli', di' ""'" onrl There are n" "ct rules_ 1n ~~~h c.a><e ycu ha,·e w
ce
mult!plioation. The problem• nf number in ro"t W•cuver tbe pattern adupt.-rl Th~n' con be
of reasomng will nr,t ruqui.t•e 'mv advanced Ollll""iun nf lcttel'Blll "n order. f,,,. "~~mple. uno
lmo\>ledg~ of ma\l,', insO;cad they ,;ill teot how ec.ch t1me.
logical you are. that i" how wclJ_,·uu rtaoon and The ea.;est "a'- to tackle lecctr ~"rie" que>twns
think while can)ing out oim~le <ltidnn~tk
ra
;, to be var,ing of L'le pos1liuu uf t.ho alphabet.
manipulaticns. \'Nbal reasoning iudude.< tOur and 1!" roBition 01un1ber 111 both l'"rw"rd and
broad oategor1e~. numo]y 'eTI~S oomp;e11011. backward •equon~ee
classif1eat.Ion (findmg the o2cl maq. out). Fur m"Umce'•

,
m

analo;;ical:relauonoh::ps nnd wiling i11 addili= W


~' B c D ..... and "o on_
other types of logi<.;a] and reasumng qu~otion.<
SERIES COMPLETIONS '
~ 25 '
M '
~3
lforwa.tll
(backwal"flJ
xa

In scrleA.lellers m- digits tlro g1ven in a &pccilic Al;o rEmen1ber tlmt to cont.inw the sene;, al'wr
sequencelm·der ~nd you have to find out the z. we "gain begin wllh A !n other \>orCs. the
next word lett.,r or rlig1\ tu corr.plet.e llio given ~cquenc~ i' kepi i.t1 ~ Clrcular ordn. There m oy
eerles_ 1'h<>r~ may be questiu11..• in which ,·ou be so~ oral .-_m: br patlerns m 1Bll€r •criea
ha.ve to 1ricntlf}• llie b,t on~ or tv;•o leLler; or Some sklppin~ pattarn8 are dcscr.Led h~lo".
.e

digits to continue the scri~s or to finO a rni>.>:ng (i) Regular order: The numher of lociK-<'e
letter or digit 1n between til~ e1wn Inters ancl akipp<•ri rPmain• th~ >arM
numbers to runtinue tl1e "'q11ence followed in (ii) Tncreasing orde~: Eoch llme th~ munbcr
w

tlle q~estion. As 1t is, thet•e i< no oet pm.tnn anJ "f lettet•s .<kipped rnCl...,aoe~ 111 o g1ven
each q11estion mar follow with a dil'fp-.·~nt ~altern. For exmnple,
pattern or scquenl ;, 1a:rrangem ont oflel.l """ or ACFJO'
w

digits_ "h1ch you have to ciPtect using )'OUt' .Answer: U


common sense ancl rea&ollin~ 'Jbllm· m. the Here, eo--h \lllle chc number of let!''T-'
q11id~"t uf your speed. - "k:!ppod "'crease., by one_
There ar~ ra.il oI)' tln~eo t'TCo uf '~rbal oerie.; (iii) Decreasing order: ICach cimr> the
w

oompbtl~n 'I uc,tlm18. <la~ely: nUJnber of letters sinppcd decJ"('~""' in 0


gown pactem . for ~xanpl~. Pure s~ric•
AOL PS'I In thio type ol numbe " a~riro the n!lL~Ler itself
An"'' er: 1- oh~ys certai n order oo the<c che eilora c!cr oft he
Here che numb er of lcttcl '' •ki~ped '~rieo ca01 h~ f~uncl oul.
d~cn'""~' by one each tm1c, that ie. iirot 5. It may be:
then J. thn :l "nd oo un Perfec t squ9.r<'
(iv) ( nterli nked g~riP.~: !"Dr t•xnmplc Pe:rfoct cube
i\D1' ,J'l.-! R'I ~rim"
Answ et: V Combirw.Lion
Bet-c. t heo"e are two inwdi nked 8et"ies. lliffer ence series
Sum~tnn~' th~re HWY b~ ropdiLic•r. of llnriN this catego ry the chang e in ord~r fur the
e.~. ;n onh,lJbc .... ., o~o
lecter 111 o~t ortler diftPrPre<' 1"-'l ween e;odo ct•noe c"tlW r.UH!b~r C•f
letter i" rcpoo td twiCE; n~xt ,et could be
Ihe eel'ies io ti1uncl out

om
ccd
(c-i) 8ume kncs ~ numb~' ~f lcttc•r s He Chan ge in order for the Diffe reneP •erie>
I. Dil'fercnce bel>~ een "'"''"cu lLYe m1 mb-_,ro is
anang~,[ in a &~l'lQf, In Lhis O~l'lP-< .<ome
-"me.
ktt<-rc; are left r>ut ~nd lhe ean.-Ji~otc' ar~
2. lllffe1·enreo kow~m, cunoecutic-<' numb ers
requi~ocl to fill ''P th~ hlanl. s by p1ckm~
ace in arithm cw progrc•sown (AP).
out nppropriat~ letter~ from th~ gi,·en
.1. DtlfcccllC'-' boJlwe~n mn>>:."-'mi,-e numbel'.S 1.s

.c
allern ativc• - For exom ple· F1nd th<'
~ perl'ecc n\Jm bor.
miosm~ lottur > of thP .<cri~t d bdrl
a_~nd_u_d.
4. Differences bctw('l'll conse<·utive <'Umber.;
are m ult1p le,, of a ll'~mbcr

ce
Answ er: eehddll
5. Diff~r~n~e& ktw"c 'n con<~cutive n11mb ors
In thJ,g '~ric>, lhrM alph,h els b, d, e hnvP
arc prime nuon bm·s_
been arran~ed La a patter n, winch lmvc
been rep~ated. The patte rn is lo be 6. Differ ence betwe en con>c,·utive "" '11 hPr. is _
discov~""d by trying ooque nceo. Only
a pctfrc t ~·~lJ~.
ra
ccbtld~ i" th~ corrod an<w~r hc=ll> e whrn
7. Dilier encr betwe en con,;n<Outive n·•mh ers is
m geumc tnc pru~r·e"<wn (G P.).
in~m-ted m sequnn ce, it rna kc the pa ttem n f
J'('penting debd. Ratio series
Under rh~< cal~~ory, th<' d>Lmge tn or<!er forth~
m
,., tioo betW€001 each cnr'"'''uLive numh~r ,,f th~

'" '"':~~~~~~,;~f:·;""~ th~


seTi"' id'o\ln d ""'-
arra11g~d in a
111 Mixed >edcs
xa

numbor< form a n
''·~ riou" ot·d.'"' of
or<ler Somc hme;, one or more 111 are Het·e. tbC' num bel'S ohe) i:tg nrc
IW~ or mnt'e ili!fcJ ·cnt t;pcs of 8enc,;
wrong ly put in t.loe numb~r o~ries. One is altc-r nateh in o eingk numb er
arrar. ged
r~qui.>:t·d to observ e thP !tend 111 which clw
oene.<.
~umbet'o cha~w· i<> th '"'i~o and C1nil mtc
.e

whwh numbe rinum ber'> mtsflt inlc lhc :;c-ries Cllanfl:e In order
for the Hati" .•eries
berv:e ,,n each c~n,,cultve nn mher ts
-.ht. rmmberlr_u "'her-; lS t.he ODD c-,l ,,ffiC R of l. R"tio
!h~ ;;nmo
lhe 8Prlf'' The c-tlwr pdcter n ;, I<· fimlou t th
w

n.!;SJtlV, or 1,,! numlw r qf a S~l~c'<. 2. Ralw bet.wcc•n ~ach <·on>ecutiv" numh H< '"
Fullowml( "re oomc of the imp om• lll ruh·< in arilhw ctic pru~ra<"ion (A.l')_
e nurnh em lS
or onl<·r Llll which thr• mlml>er snic'' can be .1. &ltio bdwe en l'-'~"X'Uliv
w

madP perfec t square numll~r-


w
... mllw uerwocn conse~m:nrc nurubtrs '"the Answers
mulnple of a nuruber_ 1_ :h) Sum of a 11 hunzont..o I rQWS ,__,~qua:; (o 120_
~- Ratiu Petween wnseC"Lltlw mm_bers L< " 2. (a) Sum of oll ,·cr,ioal columns lli eg_u~l to "00.
pnmc n'-'-l!lber _
6. ltmw ben,een mn,cclll\ve nmnhcl'" i> a VERllAL CLASSIFICA'fJOK (ODD l'ltAI'i
pcr!Cct cube "um h~r OUT)
7. ltatlOB between consecutive num lwrR are In th'" t)'P<" of que.t'on~ I'utu-_ fi>e or SLX term"
m geometric progrcsoion (G_P_) arc ~i,·en in _,u~h a w~}' Lhat one is dJffercnt

om
from the ctileL The term. which dot'S not belonR
IoTHER FORMATS I to th~t group, '--' co lied an uJd l<•rm. Candi<l:il<'.'
Sometimes the lHUllbet'" :Jro arran~ed in a fig. have to oelect thl' udd term
such a; square. wbe "''circle and \\e hm•c to ll usu~lly com~tioe"
find out eitlle1 the m1ssin~ numlwr or tile (1) L.cttBr d"'"iftcation
wron~one_ (2) ]': umheT clasoillru LJ<m

.c
Directions: !,;. the quPstic!l' !{ivren below, a /~) Wordiltclll da•"ficatwn
de[wile rdatiMsl,ip P:<l:sts i" lhe number.o
wri.ttrn insWe/ ou t,,;,de thR geonwtriwl (i}!UII'-'· LETTF.l! CLASSII'JCATION
A gl'OU]} of .alphabets or mrln·Jdual Iotter& at·e
Dot<mm:•u Ute mi.,.i ng 1<urnb.--r r"preseJJ led hy ce
a q"Rstrm> muril (?) giwn, and you h~w til ftnd tk ~et or m~1viclual
1>h1ch docs nut hol~ng lo that !<1'-'"Jl- We ean
d cnve rel"honship' on the followm~ basi"
(a) Pm;itJ.on of let.tcr~
ra
(h) Small and capir.allettoro
; (c) \c~web and 'X!llOOnent~
(a) f! (b) 5 (d_l &pennon and "l"PP1ng- patt<'rn
(c) 7 (d) l (e) lRttrrtOnnatinn
m

S<>l. (c) ;
(5x~) (4xl)~66'~JG NUiVIflER CLAS.'llFlCATION
(fio<4)-(5X ~)~9, 9 2 ~P[ In th.s type of q_uomono num hers m group of
(;<5) (~X7)~14,J-F~J~f,=o?-7 munborn replace> the letlcro._ One group 01
xa

mdncdual numhet• '" different from the rest_


@SSING TERl\IS I We '-"'11 clem·~ t·ebtionship on the follnwmg
Directi<>ns: Find IJJe miss<ng 1wmber in eu~h bt!S!SO
of the iollou i11g queslions_ (a) ~;,·en, odd m- prime numbcro
.e

(b) lntegcro, mliunn I, 1rml:unnl nltllber'


L (c) ll·luitlplt of a number
" " 75
(d) Submult1ple of a num bcr
" 00

''"
(E) FaLtor of a number
w

15
(3.) 20
(c) 15
" (h) JO
(f) Square• uf num here
(g) Cube" of number_,
(d) 3J (h) Sum ofthe number'
w

2. ~i) Dillercnce of the number.<


"'"' ""' "' 7:J
(j'i p.-,"1tion of the number' 11t ~i,-,n gruu~
w

"
(a) 71
00
(h) 66
WORDilTK\1 CLASSIFICATION
ln these que,•tlona a g:mup of Iullc, five or >JX
(c) 6H (d_l6~ worcl/il.em" are ~CLvon, une of whroh dnc<not
belonrr to tlw group. Ynu ~'"'" to 1dennty the o<ld ""'·' Larp~"-'·'"' '-'ill"~""""• ~uwo'"''" ""~e
0!l<'. t.oilor fmm R gn"LP b~cau"' Lhe,~ :·cur chan~
'The>·~ m·c ;we:ca l ha<w reiaiM "'"P' t h.l 1 ooulcl chr for~u of the ubj~cl Lv applyiL~ thc11· -hll
oxi>t IJ~(w~on word, Ho;,c'ver, a,o;nlor du~o nm b,, loL'l!,' lu , hi' g->'oup.
Some of tb.c•m at·e· So '<ailor' io ~ht• oJcllll on oul Thue. '"ilor ' '"
(a_> fi-olalloll.3hip h;, "'il on meani ~g th,ans wcr.
(h> lntor-reL.~tinn.<hip or' word;
(r) Word CUl1SlSt<:'n"" re lanon~hip r"sg<~>L~\~'E~D~R~,x~A~M[iP~L~E~H~-'I
(rl) Wc.rd fonnn tinn rolatio n'hlp
L:: TYPE-I
(<•I Function>Jl relacw01'-lnp In !hi.• oyp~ of que, tion• . rour
alten wtive o at·~ giv.,n , of whi"h tlu·~e ar<'
You lla,-~ tn deLHmine the relauon;hl~ bet wo><'n
lhe gl'·cn l<•rmo ana tl:tenLdel1tlfy 1he ocld r~rm 'in1il ar to one anoth e1· "'hi!~ the rourt h
Sone •llhot· L_v\)€ of analog Jc81 rolat.i<HlShlpe H"~:
one is diffet ·ent. So you ar~ ru4cti red to
("'' AnLuny1nou> rel<.l ion,hp : Oppo> ite b pick out th~ item whkh do~~ not lwlon g

om
meRnm g
to that group .
Lx. I_ l'ind thP ODD one ou\
(b) Synon yonuu o rcblli onshi p: Same 111
m~•mng
(a) Diff~rmt (bj.">=·l'""'te
r·elation,hi~: Bco]ogicall (c) ll1"tm d (d) filll!ilar
(c) Cla,_, ificati on , are synon ymo.
&ifi m Lion Sol Excep t (J), all c.th,·r_
n., t ~nicaL'Ph y " Ceall Cb e m i <:al cla
n. Hence, th,, an;;·,<'CI'" (l\)

.c
or S(ient ific ur Hioto tical d~ooif loatio

(J) Aruel<> purpvsr- '"latiu noh1p Cx. 2. f'out ofr.he follnwing flwu e al1h ;_,a
lei 'Tr me >equc nce r e!utw nship cet!ai ll wa)' ancl so lor'[]> a ;r-oup_ 'Th<·
Cau'e and. dfid 1~ ]a bono hip one. which does 1101 belo:>~ lo Lhat

ce
II)
1g) Wori<er >lfl:ck relalio n8hlp gr"llP is
w) Tiger Co) I .1cn
<b) Toni ohied ,-chtio ml.ip
<_<·) Cat (d_t flu,~
(i~ Who l• Dan m·p art whole re\ali011.'hip
u) Degrc~ (Jf dJlj(,r~nce rdotio nohip : !':am~
S11L Exc·t•pc (ell, ,11 other~""" carmv nraus
anrma ]. whil<• lwoac -,a heroi,.oroUb
ra
:uean wg but dlif~renl in degree only
(k) Worhr t<Jolr clot.H m;hip
amm:cL Hence tl1e answe r u. (dl
-J} Sex rclatio nohlp TYP~:.-II
;m) Fam1ly re:.atwn.;l-np (Blood Tidat w•dup ) In this typ~ of q ctesti on,, four number~ or
m
(n) Final pt•odu ct aLd raw m"t" nal word~ are giv~n, out of which three are
relation>hip alike in ;;ome mann e1· while one ;,
(o) S)'mbo li<:rol atwuo hip diff<• re!lt m1d this llumb erlw, rd is to be
xa

~'J) Pla<X',..-,lalionship dlOSl 'n as the answ er.


M Asoon atton rol.JI!onship f:~ :L Wh;.,·., 011e i" diiter cnt l'mm the dhcr
(r) Nllme rica] rolatio nehip three'!
H Spccia li;L and subjec t r~Jal!onsb1p (a) 'T.YXW f") _l..C<!OP
(I.) Phob-,o,, Jmd <hm·c cauge (c) TIGFE (d) DCR, \
.e

(u) A!;"t"Oiot.iorulhlp .so:. Onl)' in (L) the- leLtel'" ,n·e HJ bC<tuence.


(v) Comp 8raliv e r~latiunehip 10~ 4_ FnrltheOJlDurr~cul
(wi l!abi''" ]QliOJ Fh>p (a) PQR i_b) }1'10
w

(x) Qt.aht Ative or qu,,nL HatlW mlat:o nshq; (e) TIDC (J) TU\-'
(v) ULilil Yl'Pia lwnoh 'r Sol. EV<'po (cl. m all othors . the lhr~e
A8 far- exam ple: C.1t-penter, bl<\cb m1th. le<te" are in O<,<:Lence. ll<•nce, tnc•
w

~ulJ ,nnth. sailm, on rl tailut, LLllb·"~'· ;, (c).


w
TYPE-I ll <pecilic l'lllcs whid! arc g~nemlly m~nlion~d · n
In this type of questio ns, four or fh·~ the qne"t1ons m che form of exarnpleo. Sludcnt o
numbe rs are giv.,n of which thr<'~ or four are i'~quir<>d Co find the ro<lP M they l1m·~ h!
arc alike in 3ome way, whik one ;,
differe nt. Theref ore, thb odd numbe r io
:!eoode the given coded ""r.-l
Exam pic 1: Jr CEJQ '" ~nderl as X\'\(.1 then lind
the requir ed answe r. the ""d~ for BDIP_ •
Ex B- Fourof thcfoll owrng& veorca hkerna Answ~r; l'WRK
certam w~y and "" form a gruup. J£xp]anal.wn: Here thh fi'Pt. I~ lettOl'" r.f lh~

om
WhLch io ono that due8 not hdo<l~ tu the alphabe t ""C coded bv 1.3 lcoter' nf ,dph•~elm
group: t<veJ>e_
(a) l.J (h) ~l; A B C D I·~ F G H I .J K L M (l-'1"'t I~ Letters)
(c) 39 (d)51 ZY \ \'-{VlJ '!' S RQP 0 l\' (l..ast 13 Idler•)
Sol b;xcepl (d), all othe1'S a,.., diviSJb!e by It lS oLviuu" from llw above thAI ''"'lmg
13. scheme i'

.c
Ex. 6_ Fin(\ the ODJJ one out_
BooY ,D=W ,I=H,I '=TC
Ia) 4:lli (b) 427 ThE>C are various t~pe> of cod in~-
(c) Zl'~ (d) 1R7
Sol Except (h) lh ulhcr number ,_ the
middle dig1t '" tl•e ditl'c'rence o:'. tho
other two_
ce
ANALO GICAl. LF,TTE R CODIN G
The so codes arc ho<'~J on tlw analogy g"·enm
the quc,t:w n it,etf.
Exampl e: TfV!TAIVJTN 1s cod~d as Vv'X!'JBCXL
TYPE N then what is the co~e [or !I,II ~A?
ra
In this type of questio ns, some pair of
llmwer : CA'LB Th'" follow; acom•drng to th<•
words art' given, out ofwh1 ch the words
in all pair. except s one bear certain rule (hat .\-II!\A '" made fl"f'm VITA\·1 L'l, Sa '"''
~ode Wlil al'o be made f>·om the ro<le of
relatio nship. Theref ore, candid atPs are
asked to dedph cr this relatio nship and Vll:~ITK.
m

choose the pail in which the words arP CODf'\1 (; \\1TH SPI:<:C!FIC PATTER
~';
diiTere ntly related . Thus. the odd one is Here ldter are allotted certain values
hut. w J lh
th<" answe r.
xa

spec1fic pattern -
Ex 7. {a) Leg , Lame (bl Tongu~ , Tasle Fa!' Exampl e: lf HIRE i.< cod~d for a oecrct
(c) Ear; Deaf (e) Eye: Bllnd maoaag e to be E!IQJJ. how i" DOl\"E ~"" be
Sol Except.. (b), m "II •>!..her ;:n>up,, a part of ooded?
the bod.v i> infected w iLh a d!sahili ty. Anewer: H~re accordi ng to the pattern the
.e

Hence. the answer i> (b) pr-e6dinR letter'" taken f~t coding ae E for f'
Ex. H (a) .'daha,· ira: ,Tainism H tOr I rmrl <o on. Thoret:)re. the ooJe w11l be' C
(b) Chm1drag-upto _.\lacuya forD, 1\ for 0, M for N and D for e;. So tho code
(c~ Kamsh ka: Kuohru1
w

will be CNMD_
(d) Bobur: :'.fugha l
Sol. Except (a), in all other groups, a CODl:> !G BY REV~;RSING OR
ruler haO. founded hio dynaocy _ INTER CHAN GING- POSIT lONS OF
w

Hence. the anowcr is (al LF.'f'IE RS


e~~amplc: JfYOl-N G aDd AKASH are cuded ~s
jcODI NGDE CODI NGj Gl\'1.10'1 a:td H1lAKA r-Oon what will b" tho
w

I!J llri o t:ope of que"t1M 8 letter oc numb~'" are tude of AI .OIC'


allotted cei'l.1:n ,-alue• or l"<'pre>ent cena:in c'l.n.<wr-r· KOLl. J--1 ere Lhe mde i" the rcve,·se of
othet· letter, word or nurnhN accoxdmg to tho gwen word.
C'ODI:'>IGWITH NUMHERS Dirl'ction• (Q J-5): Tl<ad !he folinwin!{
l-I~te the lcCt<Or> are all ott en C'<:'rta in numaJ ical in[ormatin>o cure,'ully r>nci anowcr !h,
volueo_ qur.,tions below·
For ~:~arq..te. IfYOUNf: i.' coded ao i), ALOK A J'am1l~; c:m""h nf "x m<>mhr> P.Q.ILS, T
"' 1 then what w!ll b~ th< cude of ""'l U Thereao,IMlm arricdccuple, Qronn
i<,llliCATI0:--.1' enpncer "nd Cl~ fother uf '!'. lJ ;, Ll.~
Answ~r: 9, counc Lhe rJUl!lbC1' of l~ttBr' in Lhe gramll'atlwr of R an•l _sa ,oonlradet·. S 10 the
g~ven wurd. grandmother nt 'l '"'d.," llnU<<'Wlf<·_ There io
~ne doctor, ulle contraelkl', one 1\~r;c•, "''"
OTHERFOIU fATS hou"~w ife ~ncl two stud~ntc in the fam.Jy,
gome oen"enco" a~~ coded h~low L 'Who i,o the lnLsb~ncl of P''
Ab"h r; gum~ 'IIi> lecisjonta (a)R thl L
Arm is wo,·k1ng Kin ,i<m<.a ~1ck (c){j idlS
/1.1\l' w~' going Ki.tJ minto loti> (e) T

om
Aka,han1 Arulardri~nd .\·li> :cin kik 8H hi 2. Who is lht• ei,tc-r -ofT''
,\ce<mling to thP ab-we pactern whM '" I he code (a) R
fOl' o;om:c" (b) c
!lnew"t': Lelie. (cl 1'
Fur Lh~'r type of qUe8twno you ha;e to re.,.-J 'he (d) lrllormction inoul'licil'nr.
coded words al\d tl-.oir codes ver)' c•refully _ (e) !\one of these
Careful an" ly•i.< of the above fotm~t giv~s us 3. Which oi' ,he following can bel"~ pmr~"'wn'.'

.c
Akash = }1i', Acul = Kin, l' - ,illltta , ~oing = \•) Doctm· (b) Nurse
letr>. 'NUt kin~= pick ar oiil, was= mimn {r) Doctor or \nr<e I d) llou,ew cl€
(e) None of Lheoe
FAllflLYBLOOD RI;T.A TlONSIIIP

ce
4_ Which of t.he following are two mrrrried
Th~re are vanou~ ,-dationshlp b91Wo<-n hmtly
CG~p:o.,-1
m~nb~r~. 1n thi> I) pe of quo.•t:on you are o;;kr>c: lb\ n:; QP
(a) US, \~T
to lind d:~ r1ghr. hk·mi oJ· family r~lntmnehip rd} US. RP
\c) 'J'S, RC
be',W8Ell <Wn p<>r><olJ>,_ (e) X one of these
"l'hc 1mporlant rehtion,h.ips arc:
ra
V•' hld-t of Llo~ tOll owing io detinindy a group
The following n•:atio!lf would prove iwmeno~Jy of male member,;'
bendirio l w jon to~ 'olvmg ouch qu~'tinn>: (a) Q!T cl,) QUT
Mnth~c·s m· Father'" >On: Brother id) LIT
(e) QUP
Mmhc.-·" or FaclJdo doughier : Si,tct
m
(e) N-Jne o[ chese
1\.Iot hds or Fach••1·'·' brother : l' nr+ Ans. Q , the Doctnr, i,· lh& fa-h< r ofT. ,S tOe
.,.lothds or Fnth·-'r s S!ster · Aunt Home" 1fe, lS rh<• gtandm,,llw:· ofT nnd h~nce
11-!uther;; m· F alher's J:lothcr : Gmndmather the mother of Q_ Since 1°ter~ ~,-~ on:y l»u
Mothrr'.< or F atlwr ' f,ther : Grandfctl her
xa

mam~d collplc;; . on~ bung t.hal of ll- the


Son', wife: Daught et ·in·law gnmdfalilcr of R. i.e. U, mu _,t be lll orricd tu S.
H us Land's or wil'e'' 'i>t~r: Si&e~·in·hw 'fhll,, R nnd 1' will be buch ch1ldren uf Q ond
Husband'" orwUe'o "'"olhet: Brother.in-bw the>P mu.c b~ th<' .<indents. So, l', wh·j
Bmthcr', ,un· N~ph~w r<"nains, ohall bB tllr wtfe of Q ond she alono
.e

ll10th er • ,), u~hter: N"'"e can h<• the llur.e_ Thno. U mll'l bco che
Undt• ot aunt's ""'- u, Ja11ghter cnusm ()flntrac:or.
Si,ter's h11,b,1 nd: Brother m·law l. :c) Th~ h uobanrl nf P will h<' Q
llroohrl: o wile: Si&~Pr·in.bw 2.:a) Cl~orly. I{ am\ T Rl"<' <·hildren c-fthe •~we
w

C randl'ath~r-, onl)' son· l•'"t~er parento. So R WJ\l b~ Lhu 01ster of 'l'.


f;tF ndmolher's only .<on : Fa1.her 3. (bj P [.• I he r:ur;~.
Garmlfathcr·, ''""·father or und~ 4_ (b) 'The two rna t-ried. ooupk' nm Q. P and U, ~
Grandmotl1d " "on: Hmhcr or um~e
w

5. (~) Cloarh-, l'ot c<rtoin the rnal~8 are q, the


Crandfat lwr'o only d~ughter·m-low: M-other falher, and U. the gran:lf$1 hP"
Cro r.drnolhH's only d;mghlet··m·lo w· Mnth,,r
w
!QUESTIONS!
!Typ e TAl
Din•l' (,i Oil>. 1 ·n '"J! ,,~~' :h• joflm1 rng senes
(d Di10, fJJO (d) 1<:'8. Fll iJ
b.·-· c.'uuc·"'~ t.',,, i>".•'l {I"Jtn """'" II lhe 15.
all•"r!.C>Iirya p ,·n lo,./,n· ,•ctih <JUCS/ion?
'" 0 -n-b bo
a"oh
" ('> !mba
''""''

om
1. Hf_') -].JJI ',J.•'·: •.EW ,GLQ INS,?
abha !'> boob
'"; rr .11 Jbi I'J.o 16. be .. ... 00 ..... aabc
lc' FK•-1 lrl) b'IL (o) •bab (b) caab
2. "\l'?.Ll'. l qYNl l. EK.XPP, O~WRK.'
(a,< f.:V'J U
(") Tl'\' 1'1
JhJ .ll~U\'T(
id! Klll:V.J
17.
(I( oAAI
,.-- ..
In) hbab
----
"
,, (d) acac
... bee .... bon
ib) ~Jha

.c
,1. ll.\1'. 1fAI' H01' J•'LJI\.
rU; 11:-.:r: (c) OAhb ld) bbao
{'oJ r·r:J 18. :\K. B\l_ IS
(r.' r·
- -'
.~ (eli CAT
zn_,_v_.TJ\Ql i>ML N \CP,\ :.? (a) ]1/[W (0; ~-f\l
' (<i! I'YXD
(o') VDE\\ ·
(0} XCI!\
(,!) J'l;\'1::
ce
(c) XW
19. BAS. ?. DC(/, llDP. 1"1<:0, '!
(.7) ),'X

·'· CIVI·: tQH Ill J'T'


(•!) );.[)/' {1!; 'l'CL'
!rt) CB'T
(c) BC'I'
(b! ABlt
(d) Hl:lR
ra
C<) \!CO 20. P,\T. PF.:--1, PIN, I'OT 0
(d) !-'liT
{11) I'IG
" -1, F.i\1 ..\
(1)
:<J
\! "· •.
(b) I (c) I'CT
21. ABA, EDT•:. IFI, .,
(b) "PF:1·
!d) POT
s
' s"
!d)
m

l'.LI I. .. Q (a) U(Jll (111 (JIO


(rrj (h) IV (c) OHO (d! OJI.fS
(d 22. 1HZ, DCY, FEX.

" "1\.c.
!d)
! I. V. (', _ ,. 1-, ~~- ( e. •' (n) l'XW (/!) F.I•'X
xa

( :I•J (c) FEY !J) GHW


IW
(••) ~) It!)
I),
c. " 23. lJCXW. ~·r;vu. GilTS_ d!RQ
0. EV. Jq, OL. • ' (11) LKPO (1,) AflYZ
?

(uj 1'!:1 (b)


"·cs (r) ,JlRQ (t!) ].\1Ri i
.e

((') 1''1 (ri! Z4. 7., X, V, 1', R '


co ___ h,J[, h'rL (a) OK

M l, ,., '
llfJJ'
(h) vyn (c) K8
(b) )\/,]
(d! PK
(•· I 25. C-H,E -8,C 12,1-24 K-18 ,?
n. Kl'_\."'"r.qll, ~!R('
(d) axy
w

~Sll, (a) S-1~ (b) :\f-00


(u; !JOT (hi I )l'E (c) L-li{i (d) 0-48
(1'! f~OT (d) 'TO!: 26. rl-1. ~-4_ j-9. 2:ll·l r,, '!
w

]2,, ~Di; (\.J"\f - (a) n-·1~ {h) p-2.'1


(u) [\ OT (b) ~10(1 (e) q-36 (dl t-lH
co :\!1 ·~ (,/! W'J'O 27..'IMO, KQS, \'IJW . ZVA '!

,,,'"' ,,
w

,0. c_ H..J. >·I _,. \'


rh) ,. (a) DCF.
(c) ECJJ
lbj BG'D
(d) fZ:D
0 ,,1}
H. .\t-2, ll.' ~- (:.'0.
(o) ],;/~~ Fii I
lll~ '
(hi E/ Ill fl 12
2H. Pj~ct, d~ct, ~pic. flow0r. fluen t."
(a) o~l-:'f'l
(r) rl·Fnmy
(o) ,;;rty
(d; fluo~
29. cc a - ea -bc,cb - cea - c oc (r) ababah (OJ aa:.bca
{c<) cbaho 0) bc"b" OTV, QBT,
" "'"
]\,]()\
!c) badw (r!) Nuneoi'th~"~ (c) SUH ri>J Sill{
~0. mw. rmx FQY, )
(cj "m I'! SAl{
(~) HZM 0! l!CZ :14. llX,J, F.1'L_ llPK. KLP.
(I) !-iM7. (d) Knn~ oflhe3C
(c) t-..1-IR Cc) ;.:JR
:H. cba - <"b · eeL I
(1) MHR (") 1\H,"
0) cbea
(1) CO'Il'
cb'<c
(C}
32. -aa·c. b-aa-
(d)
hb a - cob -
No·,c~ftb<•et•
"'' ,-\1...-\., BYB. CXC
(1) DXD ,,, [)XJ\·1
cohnnc <e) DWll !d) ':--!oneolth~"~
(a) c.bccab 01

IEXPLANATORY ANSWERS!

om
l. (d): There a,-c l.wo oeriee 11Tlll. DG-J, CIL 7. (b): Th--,cc Jo an in<'t'ea>ing trend m rh~
and &JO. C I,((. INS. The fn-,t letter 111 g•p< of "ariou< lell~rs in t.h<• oen~s,
,-,e,.,. group of the oni~' com eo ,eft~" n ;_,_, l. ~. 3, .;, ~ ecc_ -~" q + ,'i = \V
gap ol' one ktter. i.e., B. D. F. and the 1!. (b): ':"h''"' "re ;wn '"""'H . r._ F. E. D, C
>CCDnrl and third lelLel'o Df co"h ~ruup a:>d V, T, R. P, ~- L 'The lir"l has ro

.c
!JaVc ;.h~ .<~mt· scqu<•JOce, v .. F.C! and gap but the .oec-on,] :\ao a gtlll c.f' or.<'
HJL. f:hrr.e 1> ohc "'"e w .ell th,, -;omncl ldte1· in deotend 1ng o>LJ8l',
-;eneo. ~- (b): C " l1tth from lwginni ng of etc

ce
wmw -" alphah<'; :md V i' fifth l'mm Lhe c'lfi
' IcI·' The l'lret ldkr in
~.
<OVt'I'
Suml,1riy .J i., ten< h from th<• hgmnm g
a•,·endi ng in the ord~r of.-\(_ ~r:; 'Th
;~cnnd letter m evc·ry group l' ol;o ant!(-) i, lOch frorn Ihe end aud rho
~seen din? in the orcl€1' nfPQRS 'I' the .•:~n>e n1k '" fullowr'<- in <he (nhet p&;,·.•
ra
chml Jette~ 1n every g1·c-up '" of;.heR< l'ie"-
cloocending 10 lhe Ol'flm ul' '1,\'\'v\·V. 10. (<~):There " a ~ap of u!l8 l~tter in ~,-~rv
fourth lerl~r 11l ever~ group 10 let!N o: ue gi'IJ~~. Moreuv n-. ~veL'''
aeccr.dil>~ in t.he o,·dcr n[U\J'R I' :onJ eeec·nd gru.!~ o' t.l1~ "eric' starts nftor a
m
lhe lo,;c letter in ~very group ;, gtlp of om· lelcer, '-''- >11Lc1' pel. we"''"
desc"m ling w!th a gap of one lel!el', ' c., ~'
'TJ:(P'-JL. 11. (b):The li>'"l lelL8l' of , ..,~ry grou~ '-" in
a, e. i, o u ar~ ,-owel3 a11d 1.bere '" u alplmbc timl ordc•· i.e·., KLIVTI'\0. Some
xa

3. (d):
voweUcon<ona!lt relot"m sh1p in ''""''Y is the caoe wir l1 ll ami Ill l~tkrc, ol' clw
group of the serie'- trL'Ou~ Sm·h ,, T'QH.f'f ~nd -~BCDE

4. (a): Asoign the nurnen cal v·>ille of ZT!AY, 12. (t:): Tl>e "irsc l~rt~r of cvpo·y ~roup '" th~
ao 1 ~- 3 ..j thcncompl~t.<• the ;;em•, in hs\ lellet of -.lw ~rcocdin!' ~rnu ~ "' i<h
.e

the ''"quenc e ot 4!J~ in oil c'>e grOlli"- a ~ap ~f t"o Htero in rver~ n· em her ot
Adcl the alpho heuc nunwri oal Yalue of L-.he gruup, ,-_,.__ af'~''1' .-\OG we «'U GJ.\-1
5, (c):
each l~tler :n tr ,, ~mup and chen sum 13. (a): TL.c gap berwet• H l~1 kr• i., 1~_,-r~a ''nf
h} an nrdN of 0, 2. :1. 4, -) ond ti
w

tcp th~ uwt anrllt>t\S wt.ic-h le equallu


'"'' in n lphab~t ieal orde1· -.•, 1lh
fvllr cvorvw hPr,', < <'., CWJ·: o- 3 ·1- Z~ + 14. (b): Let.!N'
6 = 31 whid1 is ~quali<J :1; 1 = l, FQH numeric:~ I riffct<:' rm md~r or2. -!.G. A
1(1, 12. l1 etc.
w

=t>+ l7 + R ,--:a (3 + 1l= ~and so an_


6. (b): TIJe 3Hl~" mnlain a the [ire! lecle!' of l.'i. (b):Th~ leLLc;·o group ~hb ic repoc.t<-d
th<' numhs ot th<• vear in cont.inuity Llll-e~ oin1eo.
w
l/l. (d): Varwus groups are m Ihe sel"iee "f ubc. alpll abets aaJ numb~r> ar.. doubling .
cab. he~ ..obc·. cab. ar.d S<l on. Muroovor, 26. (h): Ewry oecond lrrtcr in the 'rn~o coJIH".<
che Grot l~ttN 1n tho gToups is the lasr afLet· a li'P oftwo lelt~ro and <lll. nlwro
l~tl·" of th~ p 1eceding one", are Leing •quated vm. 11 cuuoetou tl':c
17. (d): TI>err• aro t.hree gcroupo m Lhe .<erie& mcre"·'in g or<Ler Au('h ao (J)' (2;.;'.
abcah, heahrrAbc~ (3)', (4( ·'tc.
!8. (a): Take um• lrtte>· from ~Mh group ond 27. (a): h-ery nCC"<t krrn in the sone; sl"l~S "ill!

om
"~" !hal there >& ~ gap c.f 3 lelt~n; a gap of on~ letter alan gwitl·. s uwe d n nr, c
cnry" her<• J.r .. !\, F.. T. \·1, ami K, 0, inlhe •cqu~nce of letter" meL a;, '\"'1-10,
s. w i:3 fur l\{1\'0, H.QS, ;, for QRB "ncl 0{) Oll.
19. (d): La.'' letter in "'"''0' group" hllmg 28. (d): Thc•e ar~ r::waninq!'ul word, wbicL arr
duwn. <.~ .. SRQP{l. The middl<' o~e !S dcrordin g lu t h~ ord~r of th~ d1diunar ;-.
incn•,sin~. ;,,•., AIJCDE "nd rhe fir;l 29. (h):1'he anangcr :wn< '"

.c
cch~. ccaL. ccha.
1<·11 r•r ha' lwo )a!l'3, BD >JNI llD ccaU.
20. (e): Th<' JO:tlddle h•ttcrs whi<Oh J:·e vow,•l' ~0. (<"): llit·J'le P"-"h nemlH•r of eet·i"·' 1n tiLs
haw on incre:l,i ng trenU qf ·\,E. I, 0.
U etc.
ce way
e D
,, H "ndoou n one gap in
21. (c): Cunoona U lli in betwsen two \'Owd' a"oendin g order.
"·ho ore alsu in incre:l.oing order :lnd y u q M '' nd oo on -three ~~P
oonsmla nr, are iiu'l't'asin~ in t.hc order ln d6LX'nd.m~ UdL'l'
ra
ofn.D, F,H W X Y
cwd so on-no go]• Ln
J.
2..2. (d)·. 'l'h r clur-1 ..<ix~h and ninl h ) ette"' an• urd< r~><·<·n~in~
m the re;·el>;<· orcie1 or Ihe alphab~l i.e., 31. (a):TIH: '"ries 13 <'h>W. cbac choc, cL·JL·.
Z, Y, X. W IH. fourth and sevenrh 32. (a): 'I hP ]ec<er< a;·e in Lltc <orie£ ufancchb .
m

letter,, a_-e ~oi ng up wilb " ga;> of u.w onccbb, aacchb ~\c.
lcctero an·-1 II. V aJHI f'ch alao h;m• a .13. (d): K M 0 Q - :'. o HSP 0f on~
gap of on~ k·tte_·, U 0 E - •\ .o gap of :i, Gand :1,
23. (a):l:ltal 'l re"dmg fruw CtJ, CF, CIH, 1,1,
xa

Kl, and the~ Qlt . .ST, UV. WX, YZ and X \ '


T - TI <> r,ap ul'~ lette!S.
su un after bpl.!tting che ~wu P'·
24. (d): Th~ "I ph abnb ;,r~ in ";he '"' ~r<e" crd~r
3~. (a): Tl
X
,,' H K -:---a ~ap ot !we> lctte-re

with a gap uf on~ kttel, i.e, Z. X. V, T. " L - H a gop o!' Lh1·ve


Mtet·>
.e

R, P, N ,) L N P - R a 6~P ~;· ol!e :cttPr.


26. (h):Thc>·c lB a gap of Olle lett~r ill a); ·.h(' 35. (c)
\Type IB\
w

Direclio n8: S<ndy ,,,


l!UJnbcrs and (o) m r"J M
cornp!d!' !he .•enrs by th~ ouuab/c alte1'nalic•es (r) ~
gic<•n af!<lin"l ro<:h queoiion s !"I
"'
w

J 13, 17.
1. '1, ~. 9, 10 ...... •• l~.

(o)
,,
" 0!
(<r)
" (b) ~~

""
(r) II HI
(c) (d)
'"
w

I R, 20. 36. 56,


I.
,,,0, '· 26. 6J ro>
" !b) ldO
'"
(c)
"
(to! 125

'""
(c) ill (d)
"

(d) 3, 2, 7. 6, ll
,). 5, 8. lJ. 21. (<,) )) 01
'
nne nf Lhe fnllowing "'"' Ue dec11JeJ, " '"''""' lo. ();' w1n ''pined t m tin< 'cry "'"~ uf onml<lh w Je;,l
Jevelopn1en<? \'.iliL !l10ir ro;prc:LI'C ~"""''mcc(., e.\pbin> LIJO
WI Au IIJUCSC i11 111i1 Oo empln)'lllOilt ,·mc.·nc1· c•t Jlll[J•c"·"" t:- pe< ·•I "1i111ah.
fb) htcJe,.,iut dJ< 1"''' ~r ccon0moc prngrc" 3{1, 1\hc.J Ll,ITwin ""'' cJ on <I-<• ·c~no_ the thought
(1·) JmpToJvcmmt -" htct<Lc~ of Jc,clop~lcn!
idJ bupJco,<mcnt -" boalth "'"""' .,., "_.,,, ""' h_·ord"l
2~. \\'hut io Lbc ulojo·C1i>e uf Lloc autl1o1 of '-rtlillg l/·1 l•,d ke.n p1"<•voJ witlwtll ""·)' doubl
th" pu».tec".' <c 1 IJo.l h< on rlwught nvcrl:ul w ,, nul " P"'' ed
Ia) Approu"ll of :he w<•rJJ "'""' 1 witl1 a ,pcc1al one·
emph"'" on dcvdopm<nl ""lion' <J) wa' HUL thcugh! iCI k W<'lihj oJ'rc>e'!UCit
IV) To [110\'C outhor'> ow1: prophec) .11. Aot'OJdin~ LU lh~ ;ioicon "' Dom L•l the JLilirn d
(' l lo display rk f.11lu1 c ol' rnl1cy mok" '' o: id h· beer. marked ly
e>[ i.lC p'>' With"" "bJCCli>c c•J '"' l""""ef"~ cocxi><l''"'

om
1</J .-\,lpl-ui,dl
gi>''"" ,, con•ll ucl!vc direc·ion to Ihe lutcrc ib) <l:ug~·~tor'""''al
(}irfflim,; (Qs.l\os. 311 tn 33): Read •he foii'"""S I C) "pat]) fc\l" Cll\0 "notJ;,:r

I'"'·''•X'' '"' \ <:arqdi1 """ """'"''rile quellioi\J 1/"" ,,,-, lo\'c '"'" lnend,hip
/aiJ,,,. i1 32. Th,· cxprc·>owa .. ,. """'"I nl Ll,,. rtllc>l-- J:lean<
Tile idea ol Jc·.·dopm<W (wh,cb i' c '"''"""""
"''" '~aliii'C' unJ the '"'ck .ho:l

.c
d1011~e) wa' ""' anc" o.w. The Greek,- h.1J pun .kr,vl '"' Lhq-.owdul
<Jver Ll_ ·1he group nt tl-.inkcr> ulsu inclt1rled Emsmll,, ,]ie
the fuLittr of l"harb Dam in mod ht·nc1m.1o. Lcma:-k. '.h 'be )l<lWcrful "' ,, < II a< rho weak 'hallliv•·
Thou~hl CIC<LliOll i, an Cllll(j aJI c'l liS COO ,JJWO)' >eaoofully

ce
gLJo" •mJ O>.'HI:lill1~!, (he guc" " cwreo·L I Jowever. •:r l :lie po"ertul 'h"l! he.:• til~"''"" in th~
poe,entanon ol the tnllh ~~ <hnt thougbL " -' dtffc'toJH ?fUCC% ot survival

is;ue :llmgcth.:r Dar" in rhough- that pmuf uJ' bi> "'"' :d) both the powo1ful anJ .be wtol< <11"11 I"''
m hi> nu\obmk. Ho oO<crvcJ II•·" .til tho ommab •rc J_\_ In enid climate·
ra
>Lill~_eltr,g for ,urviv"l. Those who were It~d) wnoJ f<:J .Jl lk ·"'""'I' c"n h·.-c
to tilc'il c•m·imoment>, ~"'''" th,·io ~o--,,1 <]U«litic> t•J i/•) non~ 0] lbC Ollllll:lb COli ,,U!\', \'0

thm posterity. Tht< """ tact 1> C:>ild -·,ur-·•·.•al uf "'' on I)' '"'"" ,ln>mat, can ''"me who ho\c
tbo fitte,t_" For ""mplo_ in a cold dn"""· nnly tbaL IJO:r
m
oninMI will ,urvi10 who I"' lwt bair on it> bod)'- ldl 001 ""'I' arc i0anJ with d1 i'<C!ih!)'

Uit·octium ({);. N"'· ~4 to J(,) : SWd) 1!1<· Ji•!/m·.•i"g 1<:bi' ""' , ,.,, ejl<ii•· w1d "'""''•' t!J,, t1ue,/ion< >hat
xa

follow il.
The· Number ~r oandiJatcs :l[)~>J1111g ami p:'s.;,o~ 1n <'<llli('CLill ·.•c ox,mmauo:\ !tOLl' '"'" "'"' •>bco> in the
~i•en yean-

1/w·m \'cHd wAPJ f :r{HI([/ o) Sinl<' Mu•~,ooh<


.e

l<w- I'<!J\cd .~p,o<ared \,lf!<Ciic'<-' {',_.,,.•J 4"['n>red PasJ<il


'\i'!l<'tlrc</ P""''"d
199U t.C52 ~n~ 7,>Q I 2_<1:' 5.054 ~()~
' Lli
-'-
') .)~ J 3.:14
7. I(,-1 111.15~ 4_tl I ~
I qo I 317
~-"""
C, 9.<3
'
w

I.S'J9
2, 15 3 93C 8.139 2.,160 >'.2.% 3.15~ 'J_fi95 _\,038
1992
199:' 5,U12 1.798 '!.~32
9,7~-1
3 .5.!b
-1.0:-'
:!5:29
9 Gl5 '·"""
1:1 Ll
I I ,.'-1;
1'2o1X
5.158
~.n~
w

I '194 4.9:5 1,650


(.'12_1 4 ..'26 I \02~ (,,4\~
2.392 9.%9 ~.'01
I 995 'i.G2S
w
·'-'• "' ~"'""'co., wo vo '"o '""v""'O•
per ·"''L.c~ of tile canJiJ.nes h1clu~OO fmm >cnli-
urban ort'a' obc lowest one'/
lol 1~91 (h) 1993
(cj W•U (d) 19~2
3(;. Fmm l'J91 w 1992 what"'"' tbc ncor<St
peJcent:lgc uf !'WUCliOLl m Lhc number "f >m.i- 39. !low m"ny women grudualcs ore self-employed'
urball c~nJiJalos who appeared in the ',G) 12 (b) I)
examinaLion (lo !be neme" 'a:nc)'l ~<) 211 (d) 15

om
(a) 5 (b! lO 40. How man)' o>0n-graJuatc wumc11 MO 'olf-
(<') 15 (J) 3 <mpluycJ·!
(e) 12 (o·)ll (b)9
Direction< IQ<. '-loo. J7-3S) o St"Jv rhe )<•lio>}WJ (c) 12 (di 21
gmph ,.,., 1 cmcfully w~l ""'"'"'' rhc qucoJ<On> rhot •11. AK,EO,IS,''-QA.UE
jioiMw h. {al IV

.c
(1•1 MW
(I') _'<)( (d) 1.\V
42. II ---+ moons 'odd'_<---- means '>uOnacl 1' means
" 100 ---------
------------------------
------------- ·divid-:',-!- '"'·'"''multiply' and ?I mca.c<

{:~~~~~:::~
ce
'c4u.lls' [hen, "htch oo>e of rhe following cllotce
is currccl'l
(a) 2>J.-4 ... r,-72llb
"- 20 ----------··----··· -------------
;=;=;=;=;;=
"'c,:;=···c;··---;=-·--c;···--;o··--;=--·--;::··· (b) 5-77<-41':'?1•1
ra
1991 ',992 1993 10041995 19!J-6 1997 (q 3-1-~1'2-73< 1171~
Yea·s (d) 7<----.-l-7''1'0-l-17 14
- - Com~any A----- Company 8 4J _ If A rr.cnn;; "nnt equ"l hi- ( ;<), 1:\ moun' "gremeJ
37. 11' Lloc lUld; iHcomc of comp.Ul)' B m 19~2 wn; than" (:> )_ l me""' "nnl k<< !hen" ( ~1. [) mean_,
m

j{' 140 cron:, Uo<n ~·hat wao 11> c.<renditurc ir• "i.> equ,.llo· {'*I ~- m"'m' "i1 1Wr greater Ll.an"
thm y.;:n'l (:f-1 ,mJ F mcam "i' '""than" ( <), then =.otUing
(u'• Rs IOOcron: (b) R., 110 uure 10 the ~i>cn b3Sic >lmcrncnl f4_, F5y) ""d {5y ~
(1·t.:R> ~~ CJUJO (<II la>ufftciml di>W
xa

Jo)_ wnn'h ono nf lbc foUowing conclu>tons ,s


(eJ Nu<~e.ofthc<c con-eeL?
38. lrth~ loWI combmcd c~pe"dllllrO of comp~ny in) 4, B 3s (/1) 4K D 3,
ll "'"'"{' 210 cwrc in I~~3 aCid I Y94, then whm (c') 4x .-\ :<< Id) 4A C).,
"'"-' il' tulol incoono m thcoc two)'""" 44. F1''" ,tuden" parrir·•pa1od in uu "'''"'W'''''"-' for
.e

(a) II.> 1215cror~ {b) R' 135owte geLling scbol<~r,hijJ SliJha ~ot mere cmrb tl"u>
u) Rsl40cruro (d) ln.<cflicicnl<bla PtmJO. Ka•ito g<>L b' nuub !han ~unu hm more
(c) [\oncuftiteu .:larks thon SuUhd. ;-..J;mta got nurrk> in !>tt" eell
llirodion< (Qs. 1\us. 39-40'1 o These que;""'" arc
w

c'lo marKs ofPl'"J" co>d Sudh.t. Who golthc k"'l


w
w
ami 5- 1_ Beoidcs c,·cr;- ttext group <bubling '" 20. 10, AO, 160, Mfl etc
ocart hy Z loss til~n ilo previous one. It otlwrwm , r""""'
lhe l'oqmred
numberwillh~l~2+lGO -~~2and:Jl2
21. (a):Lng~r.is Gx 2-" ~ ~ 11, 14~ 2+ ·1 ~32,
.~2 X 2 + .~- <2. 72 ~ ~ + 16 ~ 16(), + :J2U = 6:12.
22. (d): Th~r,, are h' o '~1"\CS 61J. 120, 241J :end 30. (d): There arE- l;v;•e ·'"""" 3, ti 15 an c. JJ, 30
:!0, 1 ",, 7'''• ctt- The terms in the firs' f.OHnd che log'""'~ x 2o G, ~x ~ = 12,
scrieo Rre doubl iH~ and in the lind they 15X2- 30,30x 2-60.
31. {a): Bach nexl nulllber 10 half ui' th~
"re bemg n,lved_
~3. (!\): SBcond t."rw is th~ p<Jwer ''oft he first prec~clm!; one.

tenn ~n<l fou:rth t<•rm i6 also po.,er 1 of 32. (d): There arc two oaws, <.~. 7t>. '" amlGG.
the thlTd on~. >in ~ X 4 x •I x 4 = 256 62, ~!-'
21. {d): E;-ety next t~rm, is a cubu of ohe 33. (<:): Every ""-'"end numbe_- ,s th reve"'o ot'

om
proocdin g term-~. • e., cl>e tn.,t_ e.g., ~1 - 11, .Ja 35 '""I ?.1
(1)"+2=1+2=.~ = -~'
(3) 0 -1 2=27• 2=2~ 3·f.(d):4 X -3 = n, 9 X 2 = 10
&!1<1 :J X~~ 2L
(•1)8 = (i4- 2 = 66 and ~5. (b):Tk1 ·e ~r~t.wu "HfJS_ !I 18, ~~ :Jli, ancl
(!l)' + ~ = 125- :J = 127 and Hl on_ .'15. ~8, 21 H.
25. (d): There ar~ three groups in Lil.e oeTies 6 36. (b): l'int i> the "'1'-'~'~ of th~ e~con<l El

.c
- M. 60 - I 20 and -386- 336 and ratwo ewry pair 'uch ae :9''- HI. (8)·' = CJ
are1:4 .1 :2andl l. 'md 112) 2 - 14·1.
26. (d): !lire. wrrn L> = l't term x 4. i.e., 6 x 4 37. (a): hroL ts the r_:,lf of th<, oecond r.umiJ~r

ce
- ~1 m fV~ry pa1r.
!Yth term '" = llncllet1 ll x 5. i e., 5 x 38. (b): The differ<• nee is b<•r ng doublurl ~~ every
5 = ~,, numbot·, i.e_ 5, 10, 20 41J. ~'I cl<:.
Vth!Hm ,=II!rd~.<>rmx6,r.e 2~x6 39.(c): li+3=H ,ll< l~-30ek
ra
3
=U-1 40.(cl); 1, 2, .l, 1'. 1", ~-~- 1'. 2·' :J
VIti". t-f'rm 10 = (Vth 1:erm X 7, i.e .. 25 X 41. (d)' Sum o[ emit one. lene i> equal t., 9m all
7 = 115 nun1b~t'" ~.,C<'pt in 3fl which;,~+~=
m
Multiplier& are in lO,e-wcn•a;mg oM<•r
of 4, .'i, IJ. i, ct<".
27. (e): D1vicl~ , U t<.rm< hy 25 ~r.cl yuu will gel
" 2
42. (a): (111' ~ l'' I. (I 'J)' = 1 (i~l. 117) - 289 ,,nd
(1 ~J' ~ 361 buc ~1 lS <.he squ.lt'" uf 9
quotient in the order of l, 1, 9, ' 6. and
xa

wh1rl-. 1" not" pnn•c number


2~ i.e., the <liff~rence between 43.(d):0-~=3,6-4o~,~ 1=">,4 2=
quotient > "w mcreaoin r: "' :r. ~- 7. ~- 2l>ut 7 ,'l "' no• ,,-qual to 2
28. (d): The differenc e h~tween seventh and
4-4. r~>= c~kulatc the me on u: \!1111. a,_u '""·'
"ixth i,; 42, b~twe~n sixth and hf;,h ia
i.e.,-~8+21-<-2~5,
.e

~6. hptw~en filth and foutlh i' :JO,


(6+4)~2-ii
between !Vth am\ TIIrd1~ 24, between (1+9)> 2-:;
IIIrd and ;econri " 18. It meano the (3+,j+~=[i
w

ditTerenc~' a1•e m the mere" <ing order


bu1(3+~)_,_2,_~
of 12, 18 24, 30,% and 'o on an<l t.h(•
45. (d): (2) 1 = s, (3)'- 2'7, {11 )3 ~ 13:Jl ~nd, fil
firsl term will be HJ-12 = 7.
= 125 LllC (~)'1 = 81. Thus 3 do·-'> r,,-,
w

29. (d): :12- 12 = 20. 72- 32 - -10, 102 n =


80. and so on. Jc n1eans differenc e< a·i'<l h"ve a pow"r 4.
w
jType IC j
Dir~ctions: T!!t. rJbjr•c:r.,, p;;ntl" c.r H. Hanrl~Ollle: !le,Ltlf<JI : H"sband .
concepts are re!m"d in .<IJm<' wa;.o, ym• hm•P /-0 (a) Women (b) \V1fe
,.,tabJ1sh the same "tlc,rio~"h<p" ,,th tho nthu (c) G1rl (dj She
!v..o objr•da. P'Jent,; or conc"pl.< rm the lmsis of 15. Waiting: !lof"t'dom :: f~~ucMwro <o
!he allemutuR s gmm below ear;h 'l'"'"tmn.! (a) Cl"o.;;; (b) Enlight,,n n1em

om
l. Light ; Sun : : Heat : ? (c) Schooling (d) Cunnmg
(a) Electrimy /1!) 1\ilxm 16. AHC: ZYX : CBA: 0
(c) Flr~ (d) Star fa) ZXY (b) l:lC..\
2. PaTrol : Cage : :'.f~n : 0 (c) A"YZ (dj XlY
(a) Hom~ (b) .!'rlotor Car 17. ADE FG·l . : lil\0 : .,
(c) l'rL'!on (d) ForbL !a) PQT (h) l'QR

.c
3. Dio~a>e· !Iealth : : l'"reduw : o (c) ST'() (ri) Pllli
(a) Slavery (b) Pl~a,ur" 18. UTS FDC : : l'iVU >
(c) Plight (d) IJcr,uty iu) Y"IVV (I!! W}-."Y
4, Or-ean : Pon<l : :Deep : ? (c) UVW
ce (d) HGF
(a) Rn"''" (b) Canal Hl. NU•1BER : Cl\"fl~1R£:: GHOST:
(c) Shallow (d) Filthy (a) HW'OT tb) 'I"SOGH
5. Butter : l'lhlk : Oil : ? (<) OGII!'n' (d) SOH:H
(h) Seffis 20. EGIK: FTLO : · l<'HJL
ra
{a) GJli.IT' (/lj GJP.'d
(d) Gmm'
6. Crin1~: Punishme nt : : Deed : ? (r) TTG.\-lN !ri) <;MJO
(a) Plcasul"<.'
(c) ~m
{b) Hmred
(d) Prize
"· l/1 ll-'l
(") 11-1
: 21.3 :
(0) ]/3
m

(<~
: o~ : : 1e : .,
(0)
7. Soldi<'t: Gt:n · · Bbcksmi lh: ?
(a) Wood (h) Sword "- ~~
(ct) m
'" 0) 32
'
(c) lmn (d) Hamwer
(<) ~ (<!) fi5
xa

B. Mondny : April : : Friday : '?


23.17·1 9·:47:?
(a) cluly (b) Sacurday
(a} 5:J (b) 5!i
(ej Augu•t (d) Tuesday
(c) Jl (dj J.1
9. Dioea.<C : Patholu1<)•: : Pl~nct : ?
24. o12:.56 ::1l0:"
(a) S1m (h) Staro
.e

(c) Astrology
M 132 (bi l-36
(d) Astronom y (c)11! (d) 1411
10. h:;l\3 : Appl~, : Monumen t : ? 25. J·JO :b.>
(a) }',.~mahnl StudcJOtS
lb) (a) lO (hj n
w

(c) Knuwl~d;re (d) Colle~€ (c) 17 (1} ) )


11. llod~r . ]l;[odi~ine , : Tcoohn: '? 26. (i) 2b 126: : 1~!0 )
Ia) Cl"" (b) Srudem..< (a) 127 ('JJ 28
w

(,) Knowledge , (d) Gmpo<' (<) 5R


12. .\1ct•)r Bike . Pe·~,ol .. Buses : '?
(a) Kero"',ne Oil (b) Dciscl
2(i. (n) 1:1:: 2'i ;?
(a) Ti
(ell

(h) 2~
"
w

(c) Eloctric1t> (d) Cool (c) 50 (dj 6~5


13. Angry: .'ii~ht: ·'!:Day 27. 8:9::G ,l:'
(a) lldpful (h) Plerumd (a) lt) (bJ 36
(c) Crud (d) Wving (t) 2fi (d) .![)
:X'+ 1 (d) Duys and T~och<'t'

"· X'- J (X-~~


(, >) (X + 2) 0! ,x- 2) ~8. Pailur~ nnd ,Jowth
(r:J ,x"- 21 C'J Nunenf theoe (a) Day and 1\ighl
Sucr~;o and H·"'·c' "ork
(h)
"
X" . 8".-'
~") lOX''
-IX'
' M :J~X' (c) .Mvt.ch ami V1ctm'
(c) t}4X 3 !"! 64-X' (d) :'-lone olth.>c
:!0. ,,, 39. Cur and Gnra~e
' " "
(o) 0! 125 6) lim~<. anti Stahl<
It) Lwn and Den
(,)
~"
I''
""
Din<.-vtions: Twu obp<'l, 01 e1•ent.s rue
rdated ir, "''"";D ay Yul! hctei• 16 p1rl•mM •,n[y
Ic) Co-...·s and Pol'ch
(d) Alloft'l e<c
t!tn! ~plwn ~C·itit·h lw.o· the ,<ame tyue of 40. Hancls and e·mger.s
rPiaiWHs.'np '"' iW!tPJ ut "ach q;.wMWni (a) H<-l\da nrlll·m !'hi T'ee:a~dToeo

om
31. J\.1"'1-.Inesan<l Pullieo (c) Sk1n and ('oluur( o', floth (a) aml (b)
(a! Knit'~ ;,nd Fnnto 41. Re"Jgn atwn ~nd Ot:fJCe
ib) Car 8nd Whc,•'_, (a) lhm petitJ<m ;<lid \'Jdlll')'
(c) l\r<•ada nd Bullet' !b) Abdl<'aliunr.nd Thrown
(dj Tron and Domb (c) IUdnop pinganJ DioloJ_ g:r*
32, Chenus try aud Scicnee (d) !b) and ld

.c
(a) P1\iniimr and Art; 42. SupPrv i•or and >Vm·kcl'
(u) IVTedicine aPd Surgery (a) 'I~ocher ar.d Supel'mi:<HH!eul
(,;) Grogtap h,- o nd Htswry (/>) Officp~ and Clerk

ce
(d) L~w and (\.Jlure (r) Debtor ;md Cr~oilur
33. C.upECltcr ond Wood (d) l~.'i·rior and S!tpenm
(n) Goldsm ith and Cold
43. !\hla11~ and Mosqui to
{h) lllac.l<sm1lh and Tron (a) Chol~ra and Waler
(c) e~n~in~er n nd Machn,~s
ra
(h) 'fo'Jlhoid and 1\ ~hu'
(d) Allofthe"~ (c) Tubere ulosis "ncl TB i3anen. a
3-l. Cla,, and Teache r (d) All ofthr',,e
(a) Collc-g randP!m <oipal
44. Cold :md Hot
(b) Hooks and l.1brarm n
m
(a) Day and Houl'
(c•) Wot·ksh op ;-.nC Forcma u
(b) .JDnual 'y un<l June
(ri) .'\.llof:. hese
(c) J anuazy anrl Februa ry
35. Writer a~d Poet
(d) None uf Lh<· zbov~
xa

(a) l'oem6 and Son~s


45. Pulp and 'Paper
(b) Proo~ ~nd Poe cry
(a! Yarn nnrl Fabric
(c! !louh and Lesson-<
(b) lrm1 >t.nd Wood
(d) Letcer< onil Wordo
36. I fouoe and Rent
(d Wood ami fUrnilm x'
.e

(o) l.abmtr and Wngee (a) (a) nnd (c)


!b) Cal)Jtal and Jntcree t Dircdi o'ls: Two t.orrns an• reh>tcd '"
(c) Trame. and Fair; smnr, wav. You har•c lo eslab/i, n the ,;ame type
or r.•)ation.<h:fJ between thi• llfrd ""J !lith
w

fd) ,\lloHl: lesr


37. Tree ancll'ut eot. lerms, thOI'"'!!? the V€<1 alierna ti}a 11'""1!
(a) Occoan and Sh1ps below:
w

!b) !look;; anti Lett~rs Ohey · Defy : : WGrl<


(c) !looks and LLbrary (o) Rest (i>) Llle
w
Cholleng e
(c') (d) Idle (c) People
'"J Real!tv
<7. Sickness . Health : : Hap'-Iless:
(a) Comfm~
(d Son'<lw
(b) }lisery
fdJ Beauty
~,
'"
,. Heart · fl1uod : : I.un~<:
(c) Wacer
(c) Oxygen
!b) rn
'
(di 1\.one o;' theoe
Fan• · ;:xpTCS 0 10ll' : H,Elli : 0
Soft _C'ponge : : Sh"rp . -?
(a) l<nifB
(c) Qmck
(b) Cut ""· (") ! , .·•turr
(C) i'oi11ting
0! Waving
(d) Hard (d) Nuue ·>flh""'
49, E:1trance : Exist : : Loyalty : ' 85: ·12 : . J3~J
"'

om
50.
(O<) l)ilihone sty
(c) Treache ry
(b) Halt"d
(U) Tmil
Ankle : Knee : ; Wrist : ?
(") ffi
(c) ~'
65. Revoluti on. C~unge
""
(d) Kon~of\beoe

(a) Hand (b) EllXJw (a) Tre~ly Peace !b) lla~p,· ·,Job
(oj Finger (d) Thumb (r) Work. llcnlth (d) None ofth~'c
Metre: 0 · L1tre : Volume 66. l{ed. Colour

.c
51.
(a) \Veight (b) Length (aj Shi:tt. Garm~nt(bj Hose !lnrll
(c) Square (d) lln'a (c) Kcy-Sdw<>l (d) K~neoftlwse
52. Orniclw lugist · Bi,-d: : Anchropolog!Sl: 67. TI1rd- 1Nmg : 1-\,h; 'I
ce
(a) Inoeots (b) '11nmma k (a) Fin !b) Gill
(o) Mankind (d) Noneol'U 1e"' (c) Whale (d) Tail
53. ? :Ike·: Fang: Snakc- 6H. ll1llock: ).jounl. ,in:: H1l8h:'
(a) Hummin g (a) ,Jungle /'!) Plo~r
(/>) Honey
ra
(c) Foi,on (d) Sting (c) Gnmnd (d) Tr>'"
54. ? : Light : . Dusk : Dawn 69. Principl e : R•d~ : : Prineipa l ;
(a) Electrici ty (u) Sl.uol<"nt (h) School
(b) Morni~
(o) Day (c) C},;~r (d! N'one of these
(d) Heavy
m

55. Hous~:Window · :Man : '! 70. l'l!ouu: SJlvcr: · : Gr.ld.


(a) Sun (b) Earth
(a) Hearing (lJJ c,.-.,
(c) Earth (c) Pl~nM (d) Star
(d) Floor
71. Ulluo. wotcr lwv~ lhe "arne relanwa hip
xa

56. '! , Coal : : t\bony : Soot


(o) Blusl1
with
(b) Rust
fa) Lion. FoNO~C (b) Germ.,. GatL~ge
(o) J\sh (d) Du"t
(c) Book-S helf (d) ,\llofthe oe
57. O : DalBY : : Pan&y : Rooe
72. Picher. wmer haw the· samerel otinn'hl p
(a) IW (b) Yellow
.e

with
(c) ~'tile (d) Violet
(a) Ca01dle ·li~l.t
58. '!:Yello w:: Orange : Groen
(b) Slov~. ~ookmg
(a) Plant (b) Ulrra 'iolet (c) &ok .]Ptt-era
w

(c) Purplo (d) Fore;;\ (d) Thermo nwter · lu•·!'{oury


59. Sword · Gup : : PLStnl : ?
(a) Bow
n. P;ayer · : : Ship 0
']'~om
(OJ Bullet (a) H'-"beur (b) !k.apon
w

(c) Rillle (d) w~r (c) F1eel. !d) "oneof'LIL<"e<'


60. SkirmiS h: War:: Dioease : 'I
74. Man: Ch1l<l: · FhH•r:
(a) lnfectJo!l (b) Doc~or (a) Bud l''rtlit
(b)
w

(c) J\.iedicinc !d) Epidemic (c) Br"nch f':am


(d)
61. Poster is ,]~ted w wall and pl:otogr.oplly 7S. Eler.ti0n . Votes:: S,·h·t1o n: .,
:c; rc• ..at~d tn (a) Int~r:iew (h) ;\pphoaliun
(a) Came:ra (b) Ftame (rj Discussi on (a) Noneofths~
IEXPLA..'lATORY AN SW ER S!
)"ire Sarn '' is the cn'e ~<"lth Ilnd and TVth
l. {c): Sun giws lu;ht . In th~ ·"'me wa'
terms"~ C'JW - ]'(;]] .
~iv~o he"t
19. (a): Ever ypal r oflet l'""·' Jn the t£rrm aT~ in
2. (c): J'•nD t 1< ]((•p\ ln]O C~ge. :'iuu ]arJy l.'X 11H - TIM
revet·>~ orde r a" c-l'cl
}\an " kq>t int<> Priom : aft·:·r •.riaL
. ond J<:R = Kl':.
3. {~): n·""-"'"'" the ant:Htym "f Hcc, llh
and "f"' here
Fre•" IHrll will be 20. (a): 'lhH c io a ~" ~ of one ktte r <'VCI
Lh~ :HJto nym of
"' th" lirol l~rm and gap •Jf twu ktt~r~
Slew cry_ :n the s~cond te nn o. ~-h~ o~ tn~ 1s en <c
4. (c): Do~]1" :he <JU&llt}' .,ro,e~n anJ th~ with lilrd ~nd !Vth in-m>,.
quah ty of Pond w'-.11 he Sho llow. w
is 21. (b): ITnd tel!n i" ha li of \hr bl tm·m . So
5. {h): Kull er 1< nbtain<X] from .'\IJik "-tld Oil orde r lo get \he ]Vt.h ter'll c]j,id< • Lhc
b.

om
nbtait>ed frnm :-Jcoet IJ Icd term by ~- Thil< l Vth •.ewn io hoI [
6. {dl: I'nni ,hmc lll JS tlw reou lt of CrCmc
In
tlt of ofch e !Tied one.
th~ <•lme way 1'1·ize io the "J"<'"c
22, (u):G X 2~ 12, li"- 1- .):,, ~x 2 =
1C.
lkr>ds_ ll"-1 -63 .
i. (d): Sold ter '"c-o Gun ~nd Dlac·~-,mitb uo-eo 23. (a): Sreo nd corm '< th~ next prim e --,uiul
>c'c
1-iommer. uftH ·,;_S o lhe nl""' pnnw n~mher
K (c): F rirlays e<>n1eo three rhyo afr.or ~Ion day

.c
after 47 ;, .';:).
"" thee muu th aft-er Ap1~l will be 24. (a): i:"ir>l tNm = f} -1 )I)~ -i2
Aq;u ot. :SccoOlrilerm= •' ~ 7 = .;G
iH~ao<"" ru·e "''-'d lerl in P,:.h clogy and
2
JJ 'I htrd tnrrr. " 10 + 10 = llO

ce
9. {d):
plan rt" arC' "tm\ icd ii"\~oro"""'Y. Four lht{" rm= i1 21 11=1 ;12
a
10. (a]: App le;, a Frui t and Tajm ahal 1s 2fi.{ c):l> ewrm =22 - =4- 1=; 1
:'>'innumcnl lilld l~rm ~ 3" + 1 = !J ~ 1 - 1•l
ll. (e): Doot mo p'e~cnhe !'vlcdwinc,, and lllnOLerm=S'-1=\l-lo-~
ra
Teach~re Impa rt know l"dge . l\'Lht~rm=4'' J-1 fi+ J=l 7
1 2. {b); \·lotu r hilceo u "'" Petro l and lluse "
u6€g 26, (i) {b): lind alld TUrd V:t'!lle are '><JUal Sn
Deice] fm <Iwv mg l,t ond JVth toriM ohun ld al~o he
13. {b): Fh·3t l.ent :' i" the oppooite of the llJrd equol.
m
tnrm '" i~ th~ co.'e w1th lind "nd !Vth 26. (ii) {d): Tho oeoond ;, the "IUa re '>f ch<•
term >. lir'".
1·1. (b): Hand oom e is relate<] \u Husban.-1 and 27. (c): 2' = 8
xa

j:! = 9
Beau 1il<>l '" rdote <i to Wi f~.
1~. {b):W m<in g l~ads lo Bore
~om ond -jJ = 6~
1\du"'ot LUll k-~cb lo Enlig hten ment . ,,,. = 20
16. {c): 1-l~te ,\ha s heen uoed in plac~ ofZ. 11 28.( d):X '-+"" "'1X -2)(X 2)0> utX' • 4,an
for Y and C I'N X CY~ry '-Vhc -xe. not. h<O factoriz~d.
.e

o
,- !9. (c): (X)" (2X) "; (2Xl ', (4X!"- It me~n
17. (a): The fltSI term hao a gap oftw o lotter t"tlll i" the do,;h le of the
he•~w~en .-\ and D. the o~eo nd tNm ttw sec<md
ftrst term cxdudm~ poWN SJJnllurly
"""'" ju•l aftac B, ,_e.. ~ap ul' two ldt~r•
hmn h Lerm is twice of the third tenn
w

"I herr· betw een G and .J. 'I he \hJ•d


excllHlillg row Gr.
and [uurt.h term s al~ oomP 111 a
''-'nt inuo u; "'"ie o ur.dffi" thH sam~ rule. 30. (b): The .<econd cerm is (he cube of the [lrot
term '' utl four\ h t~nr_ is th'o cube• of th~
w

18. (d): !;t. c~,·tn '" re!at~d to Tllt-d t<-rm when


thiro terw
,.,,a in r~veroc' orckt· n> S'Tll . l.."'VW.
w
..,L.,u,,r<W.J.e" maKe tn~ mach lnc" lh ardor 5{). (b):A s ~nkle is th~ low~rpanofknce sc '"
when they are rlw·rl. In tho "arne wo,_v the w riHt wit.h ellxJw_
whee ls make th" oar m m·Je r when 51. (b):L eng! h,. the uwi of metre_ ln the
they are attaeh~rJ wilb_ same woy tLe umt ofltlc e 1~ vohun~-
82. (a): Chem ietry !B a br"n ch of Scien ce and 52. (c):
Ornitholo~isl st!ld ie' almu t bu·d 'md
Pamt ing is a bron~h of AJ"ts. antlll"U~ulngist •tudr e" abou t m~nklnd
aa. (d): Func tions of vnrio u• prnfe se1un als
(a11d cc.UUTo).
ha,-e hoen given here_ 53. (d); Snak es -use fang Jnd Bees uoe ,!;ing
a4. (d):, \ll are varb us profe"':onRlB u.smg

om
when atklc k.
their pl~ce" uf wotk . 54. (tl): Term o are oppObJte to each och~r-
35. (b): Prose '" writ! €n by writer~ and Poetr y
~r,. (b)·,As windows &!'e to see outsi de frnm
i" oomposed by poet. . the
36. (d): Rent is taken on hou>~. In 'ho same houo a "" rs the c~se w1th ~)"~> and
man.
way mter est is paid on capit al, fair L."
56. (c): Thes e ore U1e fonn8 uf c~rbon_

.c
paid on usrng train de_
37. (c): ~'ore•t< have t1·ec8 and hbral 'ies have 57. (a): Thes e are ,be type" offlo »e"'
.')8. (c): 1'hc;;e arc dllfer~.nt cc•lour".
book<.
3&. (b):S lowth cau,e ;; failu re aud Hard work
59. (a): One pmni t.ivc weap on is pmrr-rl W1lh a
ce
lead" to succe ss. mode rn weapon_
39. (d): C~r;., kept in GSTagc. In the same way GO. (d): S.k.irm1shes lead tn wor and disea ;es
Lion lives in den Cow a in porch, Hors e moy lead to cp1demic.
m "tabl e, ct.c.
61. (h):P uoter is paete d on t.he wall and
ra
40. (d): Rand o have fin,""el"s fed have toes and plw~ uwap h :s fram ed
head s have hairs_ G2, (c): Hear t pump s blond wlule lunr,s pumpo
41. (d): ln resignaTion the officers to be gUll. In O>-y~ en to diffe rent part~ of the body_
the •amo way when abdic,otiun is doae Ga. (a):
m

throw n 19 lo be left. Mort over 64. (a): Frrsl io the doub le - I of the 8emn d
kidna pping cause s dislodging_ a umber. Same patte rn 10 there m llhd
42.. (h): Supe rV\so r super vioos work ers and oo and IVth numb em
65, (a): Revv lmi-o n can• t• f:han~ ~nd T:rE~ty
xa

'"the c~se uf afflCer and clerk


13. (d): D1s~"'"" and their ongln "rc ;(i,·en "~"""' P~ace.
here- 66. (a): Red rs a colou r and oh1rt '" a garm ent_
44. (b):C old i' relat~d w Janu ary and Hot 19 67. (a): Bird""~" wing e to fLe whrle fi<h uoes
relau -d to June . Fin.; to s"·im m tl1e wate r.
.e

45. (d): Pulp is 1e<~rl as a raw mate ri:il for 68. (a):
maki ng pape rs W<>Od and yarn are 69. <~>=
used as raw mate rials fnr fill'nitu:re 70. (a): The lrght oFtlle muon " of"' lvercolmu-
w

and fabri c respectively_ Whlle that of sun i;; of gold on colcu r.


46. (a): Obey 19 lhe opp= te ofDo fy and work 71. (a): One grow s on th~ uthor .
is the opp001te of Rest. 72. (d): Fi:r,t con!m ns tlle ,;ccond ~-a :natc rial
47. (c): The termo are reb>tcd as anton yms oi
w

object.
each other_ 73. {c): Grou p oF Playe r• 18 oalle d \e~nr and
48. {a): SoftneS.'! i< the clt;,r acten shc of spon ge group of Sh:tp~ '" called fleet
w

and sharp ness i" the chara c\eris tlC of 74. (a): Child grow s mto man "-'!d bud p·nwo
knJfe. intof lowe ro.
40. (c): Teun o are oppo site to e~chothor. 7~. (a)
IT.vpe nl
lli r.•dion: Fuilolun;; quest1.on s rtr" IJ=ed M ~
Co)
'"
or, m"tnx. FcnJ ihe (!G/u» of x or 1 iH each
matril frmn the chOW€8 1~<'0" hPhm each
I.
(c)
'" nw CT
(0)
"
4ucstwn.
u
"' ''
ffi' f-{,;
I. I
'" !X I m
2ii 32
'"
ffi
(o)
"' "
m
"m '
(ol flti ') 5~
(c I ffi (d) HI

(d)
" "· 352 )I-

om
(c I .H~
-'
.>82 J72 I
). I (J~
79
115 "
I I"
4J2
377
'102 Ill
') ;376
J(i(J 177 1~.)
'"'
(c) :157 !") 387

.c
(ol 107 'I l:ll
(C) 12~ {!{) 13:1
•• 110 .-so ICO
!00 I 740
no

ce
150 71)
' " no m
I(

(o) 711J and 660


680 710
0! 700 and GSO

"
'/1) (d) 660 and 70G
I (!) 7RCJ and 690
(b) eo
(ol
'"
ra
(c) II (d) 10 w. I

) 701 n~ 7\:i4
n ' I "
IW
n n 187
m
~Ol [,22 51i4 w
(o) I (b)
I 37f> 41~ (•) n {::/) n
(o) 3-'i~ ') 353
xa

(c) lei (d) %5 n.


0. I r:r TIC
CD DF Fll (!)
-~
.e

m 1<1~ m (d) no
(u) cc ') co
(c) !I (d) EG ~ !
12.
"' '"
w

I .)

"· 30'" .%

'
~~

15
3~0
" 4
w

~J 15 ))
(!) 1GO
(!) :J30
" ""
(d) Noneoftheo~
w
20. Jnoort th~ m"'"ing flgur<> m
" ' ' 0
I;, ~1~ :011

" '"
''
" 1.5 70 20
u '
!
.,
(c,
(c)
"'
~!
(d)
" h; ' (b) lG

"· " w 2~
lc)
"" (d) 70

om
~~
" ~

" 5G '
21. :)~
141 ~6'-:l ~~~

(c) 64, 100


(c) 210.110
(h) ~1. H1
N) !'on~o!'lhe>e ' '" ''
.I

"' "
(<>)
"

.c
15.
" u·' ' :l')
(c) ! (./) I\'one <1f Jht'>C

" 4~
"
~

M
"
(c) ffi
~~ (i3
M! c.-1
ttce '"
' "
' w I!

(',\ ~'){
I'
" " '"
' FtLJ"
1"· (c) I (?)
'
ra
I" (c) I (d! None o!Lheoe

" "
(c) OQ
.\G
(iJ) QR 23. w Hi !
m

(c) QM (d) :WI I ' '


"'
]5()

(c) n
:-;!:1
"'
''
(llj
xa

(c)

(c) ,\L M XL
' (d)

(r) .\·1 L (d) KM


'"· "' '' JG
w
.e

)
A D
'" y
" 42

' " (c) Y.OIJ (b) :J\X)

"
w

H 2110 (d) 151}


(I>)
(")
(1') 1•.' (d) "
y
w
"· )
" 206
w

"· H
I !
)
0
!
)

!
0
16 "
:!'JG
t)5fi l
G663>;
w

•),) cr
<(
'
)
'·' '
(c) l:l 01 II (<) 3906Ctl (OJ ');~)62;,
(cJ I! ('! l2t:\C
' (t) (d) "'Tc·ncoftho<c
IEXPLANATORY ANSW ERS I
1. (b); Numbet 1"mn1 ~ w 60 oro mu·eao1n~ Se('onrl row=;, + 1 = 'i x ?.0 = 2!0
with"" equal iHI.,·rvol of7. 13. (d): I row" (l)' (2)2 (:.lj"
2.(c): 7'J+ll=~G.90+12=11li,l0:?' 1."1 Jl l'OW = (2) '• ( 2)'. (."1)-'
-no.1J. )+ll=12 9 Illrnw- (3)'.(4i' (5)'
12~+15=1~4.144+16= 1•:oamlso H.(a): lrow=flX4=3~X4=1H
on. lir{ll''= 16X 4 =~"X~ =2:\€
3. (b): Add+ 40 and snhtraet 4D ill columns. llJtu"'=~f> X 4 = ~00 X 4 =•JOO
1=0.~ 0 -1=:J,.~ -l
0
i.e .. 11.1 • .JO = 150, 110- 10 = '70. 70 15.(b): !t·ow=l '
+ 40 = 110. 70- 10 = :w ,u,d s~ on. =H
\-lorco•el" "uhtracc 10 fmm ~a,•h mw ll"'"-·l" -1=L>,3 "-1"'21 .6' 1

om
wh1"h ie eqnul t<> th~ next n1w. =O"i
4. (d): II culurnll =! coll!ml1 + ~1. Jllmw~ 7'-1- 4~ and soon.
Ill oolumn -11 onlurtm I ·I~ 16. (s): O"e lHt<>r h"' a ~ar of~ letters. i."·· C.
Requn·~d "-"lllbel' F. l, L, ..... and the s~r·,md lellu hos a
- H8-4~- J76- 21 "- ~,Jc;. gnpu[thr ee l~t.lel,, i.e .. A E.l .... and
5. (d): 'T!:erc '" a gap of one IPrt<""' m "' Qrt

.c
coC<e!Ul", i.<. 1 7. (a): [)1 ot" uce colum ny, lw 10 7. I l. l S, ''· fl.
1-l=G.l >-l=Ea lldK 1=1.1-1 7,an·ll,2 ,3.
~G 18. (b): Tl.nw·wm• d10tancc "•JuareB ncr~'"" by

ce
6. ("): 3G = 1,) +20m eoch ""lumn a11d row. 011~.

7. (d): First IMt<-r ll! row" "1'1' in incN~,m~ 19. (a): ~urn lOLa! of '"'"h ww and column '" ~;1,
order and 11, pau· i> 1r <1~"1'€'\Slllg 20. (a): 15 (30.'~) = 22-'i. 7 ~ (2012).., 70
uJ'dPl' vorticaliy . ~X(812)=12

8. (<:): -10 anri thet•-J.\ in"'"''" 2J.(aj;( X 8=56. :J X 8 = 24. 4X 8= 32


ra
22. (a): Arlrl the fust lwll munb·'l'> ro~>·'v;,,,
9. (s): +20, +30 lne<>hur.n~ "nd -20, -3(1 n
rows respect:w ly. i e .. 710- ~0 = 730 and tlu=n ded<HOI. l.he llmd numbn and
4 30 =760, 710- ~II =690. }'UU Wl]! g<" fourti" number' e. '.C' + 'J)
Se~o!ld -1 =1'3 (8+3)- 1 ~Wand II~+ ll)
m
10. (b),Tlurd <"<>lumn = Fir'! x
column -5 = 18
11. (c): I~ x 3 = 3~ X 4 =Hi 2~.(d):J5 X 2 X (7-2) = 1,)0

1!.-lulupllP~< "re in inc«'aoil!g ord~r n[ l0X~x(6-2)=80


xa

:J. 4. G. and 4. o. 6) 7X2~(7-2)=70

'fiX4=6 0X.'i=3 00 24, (b):20X ~X 2=80, j X.) X 2= J2, J,'jY \(1


~Sx~o=~l!x&=54U x2~:wo

]2. (b), F1rs! cnw = :J:J6112 = S- 2 = B 25. (b): N ~wb<'rs "re bcing squawd fmm Mt to
.e

·rhirJ ,.o"' ~ :no/:>5 = G- ~ = 4 right.

IType lilA I
ouid co Sed•. '·Ywr mother lS the
w

1. F\,ntjng nul co a girl a mnn '"id '-:\Iy 2. Ra'~


U11de io t.h~ "nclQ d' t h1" girl'~ 11ncl~' . !low daughtn of my gramlnw thd'. lluw a1·~
jo the man r~L•llt•d w1tl1 'hHI girl ' Rav:i and ::ieeta relawd ~
w

(o) BroWer (b) Fathc·J' (r.<) Uncle. N1ccc


(ci Pathm· in law (d) Cousin (h) l"ather. Daughl~r
w
(r) rmJ<m (d) Noneofth'<C (a) Dangbtc>·ond Fathc·t
3. A islh~ mot he! of D anrl (' _If C !S the "ife (b) Huebandand \\·i''~
ol' D. then eotabh,;h the rol~c1on~lnp (.") llrolher and s;,,:er
bee ween A and D' (J) N1ece ~nd Unci<•
(a) I>-I0ch~:r (u) Mothm·m·lw,,, 11. AandBai~youn~OLl<)<:>IIC lfCi,thc
(t) Grandmother (d} N<m,clt~c"" father uf A !JuL D ;, n·lt tlw ,oon nH _ H n"
4. lntroduoiug a lady Ahm:Hl ,,.;,1, "llN ar~ Band C rdau•d
mother is L'le onl; Jaught<>r of my mnthH (<,) Dau,.hl c•r .1nd Fothc1·

om
in law", How is ,\h,nad reb led w1th r.hat (b) -'"lieec-and Cn1k
k.dv'! (c) -'"lew phc•w a~cl l 1 ncl~
(ol Brother (/>) Un"le (d) -'"lom• ofLhe"'
;c! Husband (d) hth~r U. A woman gomg -,,-Hh "'hey,, asked b)'
U. If S ;, l.he lm1l her nf \1, (ho sister C•f _'./ is "nnthe;· womhn ab-oUL Lh<• r<•in\10nsh1p
M, the Ll'Ulher off-';, ,J and 'he daughter h~twc~n them Th~ woman rophNL • ~ly

.c
ul S ;, 'P lhm1 who !S ch~ unc:e of,)? macernal ULcle 'md r~c· undr ot lll8
(a) S (b) M Jnalerwll unci~" the som~ h'" io JHe
!c)P (il) 1\" b<ly n•lntrrl "·ith that bor 0
ce
6. fbvi '"id '''a lady, "The'"" of your only (n) AHl!lanJ:>\ephov:
Lmt'Jer -~the brother of my wife"'_ How 1< (b) Mochc•c ·1nd Son
che lady rclowd wit.lJ Ravi' (c) Grand."'"' "nfl (;l'Rnrlmntl>H
(a) -,.,o,kr (d) \lcn<•oft.h<•<e
(b) !-lister n 1.; the
ra
13. A JS l.he mnt h1•r nl 1-l onrl C ]j'
(c) Pate:r:::talmothe.r HJ-law lnd"' nrl of C. What A 1o tu D_
(a) Aunt (11) \-1<11 h("r ib) ~i6rer
7. F io Lhe hmth"r nf A. C i>the daughter ol' (c) .\-lother-m-law (d) A~'-t
m

A, ,h~ "'ter of I''' K <.nd the bwther of 14. !\man oaicl co a lad}', ThP >en of :.-nHr
C ie J. Tlwn wno lS che lllll'ie nf' ,j? on;y b:oih'""" ih<" bt'other ol my w1fe'
(c>)F (b)A llow '·'the lady re_ated "ith Lhe rr.in_
(c)K (d!C
xa

(a) Moth~1·
8. A io the"'" ofC, C and Q are oisl~ro. R ;, !b) Chster
the moth~r of Q_ ll' A io the :ath~T ofP th~n (c) \·T,,rh~•-in law
which of the following Gpbon.< ;, ("Qn'~Ct '! (d) SlSt~roffather-in-lB~>
(a) R i; th<• moton1al gmn<lnwther of A 15. A is the brocher "f n "nd K ll i< t.l1~
.e

(bJ Pis:l1~ \bl~rnal wncle •Yi' A mother of B and E _, IIH• father ol A


(r} 'P & •\ ar~ oousin" Which of Lhe follnw1rg 'mtcmem 10 noL
(d) Alloftlwnbo1e Jefinilely u·u~'
9. "Your l'alher ld the matern,1l ur.dr of my
w

I<>J ni,lh,·hrothcrof K
father" "ai2 Ra,·i lc· Seem How arc they 10) Ai,thefathc>·ofK
related? (c) A1st.wsonofD
(a) Cou,ins
w

(d) A 1s the ~0'1 ol F


(b) ~cph~" aw:lAunl 16. ADf:DP. and fare memOer" of,, "lub.
(r) Real brolherand •istn '!'her" are two marl"!ed coup](-, rn thu
(d) N•me of the aho,-c
w

go up. A is the brmh~t· olD'' llll'knd. C


10. A;,; the father of B. Cis L!'.e bmlherof A, F JS cit~ ~reoi<lPnl o!Womcn'o i\seocwtwn F
lS the siocer of B iflVI ;,, the fatlwr c/ ,\, lhn\ ;, a Sitar "Pla~!<"r, DnM flach~lol', B', \\·ife ;,
e"tabli"h J'elation,hip bctw~cn F and C'' n'l( a 11\ClClber of tl:e Cl\lt_ Fuuc .,f 1h1•m
bebng to ,h,, .<ome bmil)· B ar.d F ~re (c) ;) (Jj 1
cullr·agur ·' m th~ duh. Tlo,•: lo l' J'ehled w 21. Thr mfm·m,>llon gl'en ul Q. 1\'o. 20
Ll! cia '\fico that
(a) Hn<h"ml (a) A i< rhe muther of 1:', l:l and D
(b) Wife (b) f. R nnd Dare chree children of(;
I< I Falher (< I t; io the unci c of l'
(J) It io nuL poo~ihlc to d~tcrmmc (J) All ol't.ho'~
17. On the Laoio ul the 1nformot"'n gwen in 22. On tile bash oftht· gi,·e•n Jata ill Q. '.Jn. 20
Qur,,lion Nu. 16 11' 1:1",; moth~,. wa.o .\', e"cablioh !hot who i' the• wife of ll
mother·~ c.oughl cr h'"'' wa" A reLn.ed to R {<1)!'"
01 e
(a) "'ach~1· (h) ~H~r (c) C~11not b~ rlFtr•rmJr.ed

om
!c) l:lrothcr (J) MaPrnol Lnrlc (d) A
!!!. Leela. who i.o Saban", daug:>:er m'"Y" 2~. A Luld l:l rhor l.' ,,, h~> fachd, n~pn ew IJ
L!llca. 'Yonr mother .'\.lka w the )'Glln[er i.< A'" corsin ['111 nnt 1br• bmther ol G
£i.<Cc~ 0f my r"at her. "'ho is Lhe c:urtl. dnld What rdalwlt>hip " there· h<•twe~ll D and
.,f Raja·· L'tnohsh lhe rdatwn;hip (;I

hct\vc>en Ra]e< and I "llt.i\ 0 i") h1thnr (b) ~wt<''"

.c
;a) t"oth~,. k) .\'loth<-r (d) Acmt
rb) Unrle 24. F i> the b>·c.th~r of A. Cis th~ daughtn of
(d :\-lHLernal grand bthN· :\ K 10 the >l'ler ofT'. G i" :he broth r•r of

ce
(d) .\oneoLh~>e C. Who i.- lhe uncle ol U?
!9. (The mformalwl! gi'"""
in (~. 'lin. I Rl (a) F lb) K
R'"o\-.ill.l>the 1·e lntJOn>hip bel ween l.eeln (c) C (J) Kon~ofth~>e
a'lcl L~lita"l l'n•nling to dudlw R"njan said 'TieJ·
ra
(a) S1<;ter (b) CnnRm< mothd' only daughter ;,. my mother''
(c) SL8ler m Law (J) Kone ct th~'~ H~w" 11.onjan relat~rl I•J Suclha '.'
20. S<>. !ll~mhers of" lamli"I.'\Rl"IW "nd F (a) "''" (b) Nq1l1ew
m·o lrave:Jmp; tng~th<·r. E i~ lhe >Oil of(" (i) Brothrr (d) Ncncof!l1eoe
m
bllt l: '" nol the moth~r of Tl. A and C are 26. fi, the '""other of A. C io the daugl..n .11
mart Jed couple• EIS the brnt.Oer nfC D i' <\, K "thn 'i"lerof I' nnd G io the brothcc
the da .1cht<•r of A F lS the broth,•r of E of C thr•n who !S thr• unde ofG'!
(a) F (b) K
xa

!low many m"l~ mpmher" are there 111 ;.h!•


fam1ly' (r) C (d; 1\oe.l"<!flheoe
(a)l ib)~

!EXPLANATORY AKS\VERS]
.e

1. (h): The man.> unde and the nncl~ of ".rle 4. (d): ·\hmad", uolh'>'·lll·hw has th~ only
un<~e 10 th~ sa ow !'~"on. Do he may hH da11ghter ;o he1· doughier must he rhe
wife of .-'chma(\ and the lady .s oho
w

t.hr• falhet· or lmcl<• oil he gitl.


~-(c): CmmlmoLher-Grandmolh~r-mother daup;ht~r cf Ahmad.
- ~it~·, mol±er-Daughlcr 5. (d): Ae J and f' are re,,: hruthm'" anci tf.Pir
w

Ravt Sii" fathc••· ;, 8 w hu;e bmlh<"r 1> N, "o N i>


.1. (h): Cbc:ledaughtnof.Ahurri ,al>olhe th0 und,. o[ J aud P .
w ifr- ofD "'A 1.9 the mothcr-in·b" u' D. 6. (c): Lady's Uro: her h:"" .«'nand thm lady
w
1s the pat<>rnal avnt of that son. The and dat<l '"not sufficwnt rcRarding B
""n h,., ~ »•Mer who " the wife of Ravi ancl F
&lad}' io th~ paternal motherm law of 17.(d):.\ ond h"' mothcr'o dau~hter are
Ravi. bw\her and "'"\"'- Su \he eon of c\'s
7. (a): C and J ar<e children of·" and F ;, the si&tcr will be hio matcrnol nephew.
brollier ufA "' F 1" ;h~ unolc of(: ~nd ,)
M<o\hor
8. (a): R = :Mother. lt mean R hoe two

om
dau~hler" C and Q and A i& son uf C. A ----------Daughter
K I ~

c
A
Q
18. (r): Raja
'
' ~ Soh>n

.c
9. (b): Maternal1Jncle
Alb
Facher / ' ) . Si>tc-r
I
Ravi/ ce Lalita Leela
M Father Alb ;, the sister of Soha::o ~nd thei?
fothor ;, f!_op So Rap " 1he
10. (d): (Bru\het)C - A FaU~er grandfat:oe> of l.eeh and macend
grandfather ofLahto
ra
(S'"ter\ F - n' 19. (h):
!Iere M '-'the father of A and A is the 20. (d): E (Brother) C (}lw,bciwl) .\ (1N 1l~)
father of B. ~- an:! B (Sons) D (Daughter)
11. (a):(: ha' two clnldren A ~nd H, lf A" h" [n the que,twn ·" 1.< the w1f" of C and
m

oon, F\ muet h~ ilis d"ughtcr according their daug!:_w,- io D_ The l'O.m"-lnmg


to rho qucst1on. membero al'e male members_
1 2. (b): The matcrndl uncle of \he '' uonan and 2L (d):
xa

tho uncle ofth,, maternal uncle oftho 22. (c):£ hao no "'t<o whtch ;., ddinikly door
boy is the eam~ person and boy'" fmm the tbb_
maternal uncle will be the bmther of
23. (b):A has t '"' cou.>ins C and D. Swc~ C '"
the woman.
male so D mns\ be female (acconCin~ ro
13. (c): A JS the mother of C and (';,the WJfc
.e

the data) and boUt ar~ Lhe nephe\\ and


of D so A is \he nwthet-Hl-law or D, or
niocc of i\'o fntho,- _
Dis the oon-m-law of.\.
24. (a): K ~Si,ter) I' i.• the brother A--....._
11. (d): Lady's bru\her
G ibrother) Dau~hter I C)
w

Son- b>·othH'- vnfe- man


Son'" lhe bt·other-in-law uf man, and 25. (a): Molthcr - - - - ,
hi£ fathet' will be the fat.her-in-iaw ur'
w

man and lady'" tho "ist-e1· ofh1e fother- SuJha Do<Jghter


Jn-law.
15. (h):.\, H. ~nd K ac~ h•·o\ht·" and""'*'"'"· E
w

and D may be husband and wt!e, so A


19 the Bon of E and D both. 26. (a): K - - - e A
16. (d):Aconrding to th" dat.' thm-o are two 1

f~male members D and C of the club G C


_'__
nml
1. In a m1litary cude CAUTIO I'< 1o coded "" (a) Ql'O (h) CDP
(lACJTNO. How w11l r·ou write !vllSUN· (c) RST (d) :.:oT
lllll\ST_.\.."'ITJ? Hl. If TOUR io writton as 123j, CLBAil. lP
(a) SIMLINEDSRT A.."\"D wntter. "' ,'\G 781 and SP ~ Rll ;, '.H!CLen a<
(b) S!JliG"UUlS itATDN 90847, fmd the wJc· i<lo' C.\RE
(<.1 SMIUNDEHS"l"Al\Tl (n) 124\' (b) 4847
(d) None uj' Lhe above (c) G247 (d) ,)8•17
2. CAL.~ l\"TJP.Rio cl>lled i.~ a code "' C L..-'.. ]\-,\ Il. lv!ETA!'HP.R ;, c'iJde'l ao f,)\-f,\TJIPRE
ED R. F1ml the cude DJ!" C'llt('l; I.AR under Hm> will you codo :.:OK\-ML.
the ohllle rule'. (a) OR'\1LAN (b) Oill\1VJL,\

om
(u) l.ACA.'<IlElt (b) CR!lTAUt (c) Ol\1\ffiJ..\ (d) :;;[vne of those
(c) CLli..."\1.'\J.JE[{ (d) :.:oneufthro e a. lfl.JCIIT !.8 ooded ~' GILTI:l, fmd the mde
.1. In a e~rtain code bn~ua~c. ('IlL. WAP. forll.AINY .
mn mea11s l"ccl httle bo~. SUT. M.~ll BJX, (<1) L-ill'ic-1 (bl ARINI
1uano"cll orrangedpM , RIX, ~'AC, DIR (c) NAIRY (d) RJN,\Y
mean" pile of bo'<eR The code lor 'of here 13. 1\1\0WLEDGI•: i< coded ae !.56-'!~:>47~

.c
jo ? How can GENER 1\J, h0 ,.,,ded
(a) e'AC (b) SilT (a) 75-1~99:1 (bj ~.).1901~
(c) lllR (d) ll!X (o) 755591 (rl) 7.'1-'lMn

ce
4. lfSTllJ)Yl:-; 'G '"w>·iLlel1 a~ RC'TEXJMH . 14. In a C>"rtilm ccd<• T.m1gua~e BI::AT "cuded
How will OTHER be wnt.len ·) a~ GIDV. Wh"t" ~he code for SO!JI''l
(a) TOHRJ:l (b) ROH'I'f: (a) XS..XR II!) xssn
(c) NTJGFQ (d) ::.lollf'ofthr~o (c) X.XXR (d) '\\'XYl!
5. 111 a code ei~n DHI.M. '" coded aB 15. Ii' Hl\-1 - 41\ and DICK"' 2~. Fmd th<'
ra
6~014314. Pb-, mth CA-\1ELin tl:.e same ,.~ lu<· of CAITLF. '
"ay? (o) 9~ (b) (;J

(a) 0315114 (v) ~.;729:,10 (o)91 (eli ~,;


16. If !lOY is coded a.< AC);PXZ. Wh.!L will be
m
(c) 5~1;)61;3 (dj Nonoofthc ;e
6. Tn ,, ctJde lro1guage> ~56 lll€anR 'ynu """ the code fw LIFE?
good'. 63'7 rneono 'we are bad·, ~.'iS means (a) E:MHJEGJW !U) UiCOHI·:GDF
'goo·f a11d bad'. f'md the wde to~ 'onrl. (c) l.J\IHJGLFi l (d) Kunc·ofth~se
xa

(a)2 (bJ5 17. If Sf(J<:Vi' means P( JCY what du you mean


(c)B (d)a byJ~qv·'
7. If~ meano X, X mean~+,+ mell.n8- and (a) ~TSI:T (h) MUTS
-meaw "'· f1nd the ''allle of I r, X :l + 5- (o) SU'TM (rl) IICOX
L.c cOOed a' MPOEI'O. \Vl1at
.e

2 .. 4. 18. lfLO.:IDON
(a) HJ (b) lU cade lS nccd~J[u, DEJ.Ill >
(c) 9 (d) Kone ofthc"• (a) DEHLI (b) e;FJMJ
8. In a code langccage 3h 79G iB wnttcn ~' (:') HLDEI (d} f:F'.1LJ
w

+1887 FmJ the c~rl~ tC>r 4G8ZJ 19. lf~!STER ;, coded'" ,;3,)301, l'NCl E ;,
(<>) .'\!i914 (bj ~7191 co·led ao 84070 and llOY 10 coded ~a 129.
(c) i\.'i~M (d) JJ74J T'ind the code word for S()_\j )
w

9. If Gl' N ;, e<1rled "" HVO. futd the code fur (a) 524 (b) 92~
PEK. (c) 872 (d) 361
w
lf HARD meat:s 1357 ~"d SOrT mean" n~; orgamzrd in the some otd~J' uf lhe
2468, what does 214-18 acand, fot· rnHalo Fu:tllhe propPrl,v ofM ""'I A on '.he
(a) SCHOOL (b) SHOOT h,o.is of columno.
(c) SHOOP (d) 1\'oueuflhe"e Colwllll A Culumn B
ht a C•,rcam bn~uag~ if A io 'I> rOtten a; 2, ~OU:--.!0 om
Bas 1 a"d Cas 6 what do che figure 12, ADDRESS ~1nv
I 0, l 0, 8 stancls fot'l CRUX ikmop

om
(a) D8J:::l' fb) DOOR NET ijkt,·
H Jm~:R (d) FEED CROWN ikg<Jcv
b\11 in t.he blank Wll.:J. suitable i'tgure;. r.nowrw IJlcoppr
s 16 10 v (a)O (0)1
L~S?Dl'
(~) v (d) '\oncnfth('S~
27. f•'ind the propcny of metal C.

.c
(a! 30, 38 (b) :Ji!. 40
(o)~ IIIJk
(c) Jtk :_10 (J) 40, 32
(c) : (d) Nnncofthesc
IfDUSTioe"ll~d Alll., AIR;, ,-,)J~d ~'IRE,
V!RF.1s cRlied WATER WATER" ,.,IIPd 28. fllld the pmpert~; nf ~"'' "I n
ce (<>) k (b) l
COLDUR, COLOUR" cfllleci It,\ II\ and
(c) m (J.) Nnne of lh~"'-'
RAIN 1" ,.,Jled D (J 8'1' th~n "'~or~ do t'lsh
29. e·md the propel! v of me'lal N
liv~?
(a)1 (b)j
(a) COLOUR (b) D"C"ST
ra
(c) k ldi ,\'on~of:hese
(c) WATER (d) FIRE
30. Fm~ tl:e p1'<Jper\; r,f noLal T
u. In a certain code l ongu•ge
L Chip, Din_ Chunk m~~ns stndems (a) a ~b) b
(c) e '\mwofcbc<c
ld)
att<mcl d"'"
m

2 D"' Sunk Dink means ArJUtt '" 3l. In a cncl~ bnguag~ COME io cod~d "'
W!dcnt Xl.l\-V. F1nd the code for CA'T .,
3. Jump M:mk Sink "lean> School' ;,r~ (a! XZC to! CldW
xa

(c) Yl\-'DI (d) KWG

""'
1. Dmk Mup Chimp means 1'eachec ;,
teacl:bg
1'h~ mrl~ which ;, ""~d tor Arj1m IS'!
32. In a oet'lain corle PAN- :i I ancl PAR= 30.
What cod(· do voct SLL~~~ol for CAR"
(a) 21 (b) 22
(c) 21
.e

(a) Sur_k !b) Dm (d) (15


(c) Dmk (d) Chunk 33. If a= lo called a cm::e. circle i._, ca'led "
25. In a certam code Language puiac, a ~uiiLL io ''"I' Pd I a~d j IS cnll~d a
131 meane good and laal;· square what will be the shape ,,r a -.--h,·l''
w

4 78 means oee good pictun• (a) ll. (hJ 0


729 mean; p!ClLLYe •re fmnt. (c) point (t!) =
Which numb~·r has been u;;cd here for 34. If A:-JO'TJIER i; cod~d ae 73Q9,)2C, then
w

famt'.' THOR?{ will he n•dd as:


(a) 9 (a) 9Jl03 {I!) ~~~IR
(/)j 2 (r) %01~ (d) %11~
w

(c) data ace maclcquatD 35. If HIM m~an>


!I%_ :.nd lAM mean., 39
(d) 'i wi>at will be the code for MAP?
26. Certain meLala a c~ given in ColumnAand (n) H8 !b) !H
th c ir p ropcrtl es in C<J lumn B. But chey are (c) 4G (ai 20E
I EXPLANATORY ANS\VERS I
1_ (b): CAC Tl OK" ]l.flS U)[ DB; WJ 'f.-\ :ill 7.(c): 10><~-.'i-~-,--1
UAC lT NO= SI\f NU F.D SR A'l' Dl\' ln .<uch typo oCqllcswom a;!pC'. lb, rule
It m~~n" m I k ti;·<t group chre~ !etten< ofBOJJ~l-\.S
a<c rov<•r.<crl c.hen each pmr is bein~ I-!= Krnol<ct (_) = Gl'
re\-orsod_
D = Div'"ior, .\·1 = ~'h:ltlply
t. (b):(' •\L,\1\ llr; H=C ll:tCUL..<\.H
A= .\dd·ne S = S;;~l""'"""
C 1.. \ 1\.\ ED R=C R!UCAL!l.
!6+:J ~ "2X'I
Hece tir5t and ]agt letters ate nol
!6-'-.~-.'i~'-'-'1
rl"p~s1-tioned but other pairs are being
revel'AAd_ 19-\0=9

om
3. (a): .CUR WAP Dffi ..... (1) R. (n): 3 -+ l = •I ,_.]' che coLi~

red hllh• bow' r; I = 4 of lice ccd~

B!X FAC Dill (~) 7+1=8ofliwcud"


ptl~ ct' boxes_ 9- l ~ <~ <Jf Ll.e L'LHh-
Do D!R =be-xes 6 < l = ~ of Lhe ,-.,d~
B!X FAC nm .. (2)
"" .," " .,' " "'"

.c
p1lc rf boxes
Sl'T l\-1:\D ffiX _____ (:J)
w~l: arr=e pile ~
,,'' _,'' "' _,'c .,''
ce
So BlX =pile
'Ihusof=FAC
1. (c): STUD Yl KG "OT TIE R
""" ' '
"'
RU TE XJ MH" C..'U CF (j 9. (a): Evc•ry lcttc•· in the co<l~ 10 o,·,o erre1·
ohc~d ouch as
ra
In I he r.cst pair S iscod~d ru; R meaning
lh<>r~by" letter before S. next letter T C~H- U=V "-=0 'J'h;;l'ooQ
lS coded ae C, means a leLLer aft'-'r 1', F.= F' N=0
i."., RSTU, TCDTC utc. nrc turning oe- 10. (d): R ;, in r,·c,·y ;:r-oup or IeetH and '." m
m
ljU('llCC$, CVCI':Y J:l'OUp •)f llUlll bero ,u R = 1
5. (a): Add two in the s~rial rmmh~r ofl•·ttt-r' CLEAR=56781 _r!J
"" : SPARE=!!ll"~ 7 -- (,,}
D=1th+2=6 C=:l+~"'!i n.,,., H~ 'i " r<' <Oflm mon
xa

R"'1HLh+2= 20 A=1+2=3 So CL = !5() and :-!f' - '!'I


],=12t.h+2=14 111=13-t-2=15 ThmCARE o5f~7
,\=lst+2=3 E=5•2=7 !I. (c): Make ~<lir.< aml theii ceHlr>'<' e .Jo p.1ir.
L= 12r.h + 2 = H 1.=12-t-2=\1
.e

I.Li.,
6.1<:): Yuuarcgood=236 .... (!) ME'!'.\ l'H ER :10 HJI,I :\L
l'l'c are bad= 637 .... (2) EM AT liP TIE 0'-,' :VIR L:\
'"""' = 6 12. (a): There are two wou])'- l Jl! "nrl lH ·'~ch
w

We""' ba1l- m7 .... (2)


being reversed
Gmd and lllld ~ 358 .... (3)
Lau :1 LIG H'l' li,-\1. '\Y
c;JL TH JAR. YN
w

.l"'U "r~
good = 2M
good and bad= 35~ 13. (d): Add unit nnd t~"'' of nnnc'"'"! ,,: c.t ,,f
goo;i = .'i "o and = 8 lc<tc" ond1 as
w
23. (a): Colour obnds fo~ WATER so Lhe FISH
\i)
K=i;~ 1+1=2
N=H=11-1=5
0=H>=l+5-6
s,,
G= 7
E~ 5
"' lives in COLOL'R.
24. (a): On chc has1s oflst. and IT nd equ" · i<Jno
''otudr'nl,'" -DI'<
W=zJ-2+-3=5 N= ,; On the),,,;, llfilud awl NLh e~u<.c;:m 0
L=12=J+2=R R= 5 ''ii' = D!},'J{
t~0-~+0~5 R=18=J+8=~
D=-1=110=4 A= I SoPutting these Cwu 'alues Lillo

om
e•~uatiun lind we i;~l the code :·,,. A'"Jun
C=7=7+o=• L=12=J+2=3
t; =Gch= ~ + 0 as SUNK
14. (a): CGde<l Iet,-erB are ahead from their 25- (c): 4: good 7 = Pidure ami 2 and 9 =m-e
ongmal i~tters in tbo order uf 4, 3, ~- and faint respectively.
26. {b):By comparing the two columns we ~e,
,,,
"nrl 1 ' e.,
oodo for NET- ubi, oode for CRUX io
" ov
I
'' Dlff~n>nre
'' oue

.c
cjmv, cndo for SOliND i" ihmop.
0' '
'' XR I
,)
I X CRO"\"Y is ijktu, CROWDY i; 1kgotv
(b): TO"\! DICK ADDRESS i• hloopp. By funh,>r
" 20+10+ l:J:c4iJ 4+9+:l+l!=27
CAnu;
ce
comparing we got oodc for F.= b, "'-1 ~ i,
T = n. C = j, R., v. U- m, X- c, ::1-
3+ I +20+20+12+5~61 P, 0 = k, D::: o, Y = t, \'1::: ~ antCA =
16. (a): B = AC It ml•an,originall~tters ""'' L
27. (a):
ra
b~ inserted in
0 = r--·p between the two coded 28. (d):
;etters 29. (a):
Y=XZ SuL=IL\I,l=HJ .F=EG 30. (a):
m

31. (a): X \i I
andE=DF
17. (d): :3 K E W ThediflerenceJ y Q V
---;l >3-!+1'"intheaerie • -213-l+l ''
ABCDE FGH l J KDlli~
'
I I
I'QRSTU\"II'X \I
POCY2,3,l, l HCOX It means C =X or third from lefc and Ill
xa

18.(d): LONDON DELHI from right and 0::: Lor 15th frum lel"c
+illlll l 1 1 1 1 and l~t.h from r1ght. M =_'\or 1:1-.h
MPOEPO E F M I J from left and 13th from ri~ht. K= V or
19.(a): SISTER UNCLE Vth from left and Vlh from r1ghL.
.e

5~5~0~ R4 6 7 n 32. (b):l6+ I+ 1,1=.31 ond 16+ I+ l~=:Jo


ROY 'ON so3+l+18=22
1 2 9 5 ~ 4 33. (c): Whe~l moons <:m:lc whu±t is equallu
20. (b):i '"-. 2 ~ S, I- I!, 4.., Q and 8- T pomt_
w

21. (d): 1:! 10 10 8 34. (b): The lect~rs ar~ code<l hy nnm bP"' on~
1' E E D t.o fit>d the answe1• select the respect1ve
22. (c): The differ~nce ~el"een Sand L Ill llt.llllbers, i.e.,
w

6x:!=l2+16=2 8 ,\ N 0 T H ], H ----> letters


The ciiJTerence belwaon Land F ;,, 7309521----> oode
5~2=JO+Z8=38 So. T H 0 R N ---. letters
w

The d,ffccenco bel ween V and Pis 93013 ____,code


5><~=10+10=~0 35.(d):HIM=sx;~x D=9JI:i
and the diff~renee between P and JiB CAM=~X1XIJ=3~
also 5 x 2 = 10+~0=30 thenMAP=l3XlX!6=20~
Oir~ction•: Ce!tai" u:ordslterms "regive" (c) New lJelh1 (d) ·llinl,,hod
be/011' m HU'h quesliou. All are .<imilar on 13. J{cd. lll.te. PllrpiP, 11,_,,_ 'Jc·,nge,
n<rture except one whirh ~' rliiferen!. Pick out (a) H"'" (b! Or< l';[l<·
lhe odd on.e. (c) lt<<le (d) fhl
1. Man~'U.guava. grap~s. potato. pirwll ppl~ 14. Ey~. Ear, 1\oce. Fill~"'·',""''~'-"·
Guava
(a) (b) Pinea~ple (a) l."ye li>) :iu"'
(r) Potato !d) Grape' (cj Tcngue (d) Fi,J~ec
2. Cod and H~n Hor•e and Mare. Peacock 15. Milk. Curd, Wine, Cheese. Butter
and I'~~ ]1(n, Dog and Eioh. Cow nnrl Coot (a) Milk (h) WiJJc
(a) CowanJ.gaat (ei Butter (d) c.'md
(b) Hor~cnndMa,... 16. Iron, So<i.iu"'· ,\\oTcury, he~"''"~-. (;old
(c) Pca~"<lck and l'eah~n (a) lmn (h) .\'1,.rcury

om
(d) Cock £nd H~n (c) Sod.Hun (d) Gold
3. Lwn or.J. Den. Cow and Pn~"<·h Pig and Rlfie, Piotal. Cmmon ~.JjseJ'.e, Swo1•d
f'<-n, Hen and Fa1m, lforae .md Stable (a) l:'l•td, (b) C\·H,nun
(a) Lion and Den (b) Cow and Pon:h (c) O;lieeile (u"! ;:iwoJU
(o) P~g and Pen (d) Hen and Farm Li~at·d, Sr;akc. F'nx, Tot ~old'. Chome:€un
4. 61, .JG. 9, 4~1. 125, 81 (a) Cl1nmd"m (b) FnA

.c
(C1) 81 (h) 125 (c) 'l'orlollie (a") s,~,h
(oi 9 (d) -36 1\1. Sun. J llPlT.er, !>!non, Hor,z .~ Cloud.
li 14. 42. 49, 44. 63, 77 (a) Ckr.Id (h! l!oclzo•,

ce
(a) 44 (h) 49 (c) Sun (J) .l-Imn
(<) 63 (d) 77 w. Febn.wr)•, AptJ., lkc~n1b~,·, .lnly,
6. J I H G F. 0 N M L K U T S R (,!, Janumr
XWVUT.CDEFG (a) A~nl (l,) JniJ
(a) CDM'G (b) ONM!.K (c) ,Jsnuary (d) Febt'.lary
ra
(••) JliJGF (d) l!TI'lll.Q 21. HP Co"l. Bull, Hor<(', Lion. G)W
7. Teacher, Prmcipa l. S1 udent, Header. (a) Bull !b) CJW
Profes.sor (e) Home (d) lol(m
Festive. ("J,~e: rul, Jovial, Ln•el;·. \' oc·
m
(a} Student (b) Professor
(c) I'r1ncipai (d) Hcod~r (a) V<X' ir) Peetive
8. Crow, Pigeon. Sparrow. Bin!, Kite (c! ,],,ja] (d) Noncofthe"e
(a) Pigeon (h) Kite ll;!agazino. ,JcurMl, Nov·ol, D!Ctwnory,
xa

(c) Crow (d) Bird Art!clc


9. Wlwlc. f'mcodile. Tiger, F1.,h, Tortoiae !<>) .;,~icle (b) Novel
(01) Tiger ,1;) Whale !<) Dictionary (d) ,T<>urr.el
(c) 'Iorto1sc (d) Fish 21. Woover. Tailor. CarpentN. Clerk.
10. Sparrow, Engle, Crow, Ostrich, Kite Rlat·ksmith
.e

(a) Ostrich (b) Eagle (a) We~v,,.r (h) 'l'ailor


(c) Kiw (d) Sporrnw (c) l.ltrk (d) CaqJ<'rtLel"
11. l:lowl. Plate, Bnc.<et. Cup, Pan 2~. Cit·cl~, Ccnn At•ea, Triongh C·,!indcr
w

(a) fuwl IN Bucket (a) Cin;le (b) Colle


(c) Cup (d) Pan (<) Cylinder (,)) A1·eo
12. London, Wa.ehingmn. Rcyadh. :'-l~w Delh~ S,em,. ll.!JoLo. Frmt,, Pbn\c. L.eai
w

Aliahalmd (a) Pb.te (b) Hoots


(a) WagJunf!1:on (b) Reyarlh (c) Waf (d) St{'JnO
w
27. ,\, .11rl lh:yr~n, ~fathemalics ~nd. (c) Industl')' al!d Workc"'
f:eom•'try. ~"'lower snd Petal, Teache!' and (d) Ho,pitnl and 1-'~b~m'
Siudc:-t<, Ronl<~ & Sentence' 30. Cr1m~ and Pum"hmmt., J{xp~cJS~ and
(o) T·· nchH onrl Stn.-lent" H~alth, Judgement ond Advocacy.
rL•) B"<>b "nd Sentences HardwMk ~ml
.:('J J,'lt·WNorrl Petal Succe"". Slo,"th snrl F"1lm€
(<i) ,\ir·,ml 0xygcn (a) Slllwth "nd fo1lur~

om
28. Liun "~rl lto,r, I•:Jepham and Trumpet, (b) l·l;mlwork and Succ~ss
Sn"h and Hl-30. Dogs and Cook, Bird.' (c) J\ldg~ment and Advocacy
'md Chn·p (d) l•ix~rdse and Health
rhi (lt,-,lsanllChnp(bJ DogsandCnok 36. College <.nd Pnncipfi:, """Y "nrl
(c) Sn.li"· ond Hioo (d) LiUJL and Roar c~mmancler. lnJu,l.ry aml lhrcccor.
29. ~h1rt ond Tailor, Te~and Cuffee, Pen and PlaygrO'lllCl and playt•r, Post ofl'lce m:d

.c
I'"-'"'' I_ Swnrd "ml _\rwuctr. Buuh "nd l'ootmaoter
S La Lior." n c,, (a) Colle~e and P1·i n('tpal
(u) Shm .on.-1 Tailur rOJ Savv an<l Com :nande,-
lb) Pen ancll'el!C!l
(rj flo'Jko m.d Statwn~rws
(a) Swu!'d and .-l.rmmn·
ce
(<;) Pbygrwncl and Player
(d) >lone ofche ahnv~
37. Uncle ami Ni~ce, F,r.hcr and Dau~hler,
30. Bluk and Willi<', In and Out, Fm' ""<1 Brothe:·>Lud SioL.er F'"thrr·m-law !L'ld Sol!-
Con>. F'1;h and Water, Day and \"ight
ra
111-bw.
(a! l'rns m'd Cun" !/;) F'!sh and Water (a) Unc\• and .'itece
(c) In and Out (d) Day al!d Night ib) FatberandDaughl er
3l. Oil ,nd l.n mp, Vc' at.Br al!d fee, Wond and (c) BroL.!t~! ~ud Si,l<'r
m

Tabk S\lk ant! I' ant. Flour and Bread (U) FaLher-i n-law and Son-in-law
(a) 01l unci Lamp (b) Water and Joe as. RQS, DAC, !\MO. KLM, 'lXZ
(c) 1\'oc,d 'mtl Table (d) Silk and !'ani (a) RQS, (V) KLM
32. RcalJl.iful and fhnd<Ol,lC. Bother and (<-_) YXZ ;<I) XMO
xa

Wol''"), Cold and Clllil), Avoid and 39. 8FGH, WYZ."'-, Y7.>,H, PQRS, .\1.'-IOP
Puni"h Ho.oby at>cl Recre~tian (a) WYZ."'- rb) 8FGH
(a) A~oHl m10 ] 1\!fiJOh (c) MKOI' !d) Y/,t\B
(b) !lobby ond fkcreal.ion 40. CE,\R. TEAR. FG,\fl, WEAR, BEAR
.e

(c) ReantlflL ~nd Handsome (a) CF.-\R (I!) TEAR


(ri) Both~r and Worry (c) FEAR (d) DB.J..R
aa f>ulb and Lighr., Run and Heal. Cluck and 4l. A, 0, t.:, 1. Q
Tinw, fl!vn a"d Pund, Chimney Rnd (a_) A (b) U
w

Smoh (c)Q (d)O


(a) Chimney :md Smoko 42. BOC, MJN, TOV. WAY. POQ
(b) Sun and Heat M J\.11."\f (b! TOV
w

(c) Clock,mdtimc (c) WAY (d) POQ


(U) lti,er"ndPond 43. 9, 28, 65. 126. 12~
34. lnduB\rv and \',-"orkcto, Eo"pital and (a) l~ !D) 1\G
w

Pot1enLl, Marlet nnd Huy~re. D1oc•ase a~d (oj 126 (d) ~


11-labr.a, f'l ass and Stud,•nb 44. 1166, 2354. 6:172. ,·lH~f'i, 46';'3
(u) Diocos~andMabr~J (a) NJ(;S (b) 17~6
(b) ('!,.o and Scmknt,; (o) ~:-!72 (d) 1G7~
(o) }J.pha l~·t;,
45. ~4. -16, 48, :w. 12 0)
Gam'"" ") Meu,
M 12 0! 36 (1')
Geeta. Q·m, n. BiiJ~~. Tauml, I'anehsh~cl
'" 30
46. W'J. r.2f,, 196.
"'
(d)
l 1·1
0;
"
1@
"" (a) I'ancOohe~l
(c) GCH:i
(hi Qumn
(C) "'one ufthcs~
(o)
(c) 144'" ('I 625 55. T"v~olBr. l'm,Lll•C!Cl'.
l·~ntreptenm-c, Produce~
Landlord,
21~. J23, t<18 . 11 I ,
•j j
"'(h)
(o) m
(<) 121
:i23
(rl) 212
(<<) T'rud.uccr
(c) L"-.l!dlord
.'i6. Horse. Cu.,, Do~, Deter.
"''"'
Pinancier
No11o 0! the,.,
Rallblt. \,not
'"· '" '"0) roo
">06, 10R ~.'iO. 483,
0!
I")
76ll
(c)
'"M
4R 2:1 43. ~- 29.
8-50 (!) Cnat
(•.! .l:tabbit (d) :\one ufthc,~
~7. France. Turkey, Gt•cr·k. lt.>ly. Finland
"'
"

om
f'mroc"
"
w
(11)
(c) d)
~
)
(!) ri.1:J.m:d
(c) Italy "'
(!} 'l'ml;~.v
READING
.'iO. , TIRF.ATHINC,
WRITJNC .
·" Hal'bour, bland, P~ninsub, C~u,t, Oaols
0! 0""""
'"
·lU!viTlNG, S\'llJ\ITI'lll\G Hod:·our
(!) BREATH ll\G ~) JU/vJPT"iG (c) .Lslanrl (d) Xune oftheoo
(d W'RITl:-.IU !'! RE.'UJrNG 59. I'remohonC., Kahdo,o. .J B. Shaw,

.c
:'IIa,·o, Sb. Jnp1te1·. Mnon. Suu Shake,pea t'C, ),brio"€
"· (a) ~un (b) ~far, lo) T'remchaml (b) -1 1-l ~haw
Nor,eofth ese
(c) }lmn 8k}'
"' (t) Kalida"
'"'
ce
Monpnes e.lluhhm . Salt, Sum~, PeL.nJl,
52. Sabra, AralHa. Tiwr, Goli, St.ndarba n
""· Cold
(aj Sundarb" n
(<) Tbar
Ambia
Nune ofthese "''"' (o) Rub be, (b) (;old
(d) Petrol
" Alpha, Beta, Meta Thela, G"mma (c)
'"'
ra
I EXPLA NA1'0 RY ANSW ERS I
I. (c): Potmo ;, a vegetable FJn<llloe reol& arc 10. (a): ll ro tkonly bird tl1at t'anMt fly while
other can fly.
m
(ruito.
2. (a): J.;~ce~lllus all tO.e resto arc paic•, i.e., 11. (b): All are ut~n."l" exoept ·.hi:'
une maooul:.ne and one faminine. 12. (d): All arc the ""I' ita Is of \'al10'lS rountne8
3. (d): Llon live8 rr. den<. Same is the case excepe Allahabad wbi.ch is a city of
xa

w1lh other "nim3.l8 exOPpt. hen wh!Ch lndia.


Jive., in bu>.ea not ~t. farm,. 13. (c): Ruo~ is " flower and ~u the rc<ls nl't-
4. (h): All are sqMru> except 12'i v:hu:h" a oolo\lib,
cube ot fi. 14. (c): Barnng Tungu~ all are cho external
5. (a): H I> :h~ 0nly <li~ic which " not orgm, of the body.
.e

d.tv!s!hl~ hy "'ven.
15. (b):t;xcHp l win~ ~ll hlong t •., th, tm:k
6. (a): A;J arc m reverse alphabeti cal ord,•r
eaL.e~ury.
e~crpt. thi.; option
16. (b):lt iH Ll1e only metal which i> fouml in
Except stud(•nl. ~11 are teaching pro·
w

7. (a):
liqmci form.
[essiono Is.
8. (d): Barring thi., option all the re.,ls are 17. (d): J<:x~~pt Sword all are fire armo.
name< o:" hirdo. 18. (h): I'~~ io a hunting "nimal and. the re"l."
w

ar~ IT'ptiles.
9. (a): lt io Che only anim"l thall.Jse8 on hnd.
w
19. (a): E~cepl. colnud ,]1 the terms are u"Hd 1~ !rom middle th~n fll',t and thm I&H.
asm,wmy_ .19. (a): ,\11 th~ ,.~,~-' ~'~ '" ~lphabehcal senos
20. (d): Frbnpry has Lwu pmo.iLililie> nf2f'- and 411. (a): Il i;th .. only ~_rtwn that Uo~s not ha'~
29Jays · any mecmong_
21. (b):It i> the only female mumal in l.he 41. (c): Q is the only COl~"Onant lil th~ group.
W'>U]). 42. (c): All the grou_r' aN surrounded byrun-
22. (a): Bxn•plll11s all !lr<' che olgll3 uf pleasure "onanl.s Jn alphabetica l order excepl

om
and huppmcss. thio_ lc i" a m"or.ingful word aloo_
23. (c): Dictionary is tl,c ~-<1llod-10n of wordo in 43. (a): Alllh~ reaoo are ""heR + 1, i.e., (2)8 -t
alphab~t100 I nrd<•l', Jw9,(3)' ll=~8,(1J'+l~n5and
24. (c): Ic 1s ,ht• only profeeswna l JJn-uh·"d in :;o 011.
wh"e mlhqoh,_ 11. (a): Reasoning No. 1-8 iathe unlydigit.
25. (d): EX()9)ll Lh ;, "11 ~~r CPom~tncal fl{(ttres whic"- is repented m this gl'otq> and no

.c
wh1fe area is a unic_ repelitiun uf a digit i; tJwre in aiJy
26. (a): All are the part• "nd p~rcr>l of plants othergrou~-
27. {a): Teacher and Student-•. In every paif Reasomng Ko_ ~---All the four di~JCb in
the second word originates from the
fust word.
28. (b): 1-iere anln1alH and th~ir voice oi crying
ce every group are in increasin~ nr<ler in
difl'erent wa)'"-
4 5. (c): llnil '• twic~ of t.h~ tene in evel'}' group
~rc pa1red ~ud Uug~ b"rk r.ot cook. but her~ 1t is thrco timee.
29. (a): F.xc~]Jt thio opc.ioll all ~airs~"' wsed as 46. {:>):!Ill tlJ.e re;ts aTI' ~quaJ"<>il ot' cetlain
ra
phr"--'""- num be~-' PWepl Ll11>
30. (b): Except this terms wsed in al! \h~ pe.ir' 47.(a):Tn all the other numhnr' firot and
are opp0site c-f ~aoh ot.hcr.
31, {n): Oil and hmp< arc two d1fferen\ things
thu·<:l digit> are the ""n"'
48. (a): 1\"o zero B u~ed m 48:J
m

"-llilc mother ca6~• the s~cond word i& 49. (d): Except Hall ~rc pnmc nurube""-
ohe changed form of tk first word, r.e_, GO. (a): It ;, J.he only automatic nstm!l 1"''lwn
ice i" made o!' water, Nblc is made of ~erfurmcd by hrin~ or~anism.<_
xa

woodandeo on. !H. (d): All th other< are llea,·enly bodi.,s.


32. (a): F.~cept this alllhe pair< ore <ynonyms 52. (a): Sundarhan lS a ddta whil~ the ot.hn<
of each other_ are de,rt.;;.
33. (d): Second word is che c~uoe of the fll'et 53. (d): All the rests are Gteek symbol• U"l'd
word l"lt thi.> pa1r i" not related mlhL• especi~ ily in malhemal.i.: o.
.e

vmy 54. (n): All ilie reRt' aro rdlg:ious boob except
34. (a): Th~ o<'<'ond word in every pmr !'<'late> to l'anch,heeL
L},,, fir"- w~rd, i.e., wC>rkcr~ gn to the 55. (d: ll i" ctJ!Ilmonly US<'d in farm business
w

indust.r;, patiemo go lhe ho•_rital, wlliJ~ \.h~ othc"' al'<' rd~ted to f1Mnce
buyers go to che rnrkct m:d sc on_ artrl busl11e.es.
35. (<-):Second word ;, I. he rent!! of the fir<t 56. (b):All the otherB at~ vcgctorinns .
w

word hut judg~ment is not 1:he t'e".Jt of 57. (d): It is the only Asian country in r.l,o
advoeacy. group.
3G. (c)' Jnol!LuL-i'"'·' and therr h~d' are paired 58. (b): .'\.lllhe o'brrs Sl'e ,-,ear S""- while Onoia
here_
w

:s seen in the rloserto.


37. {c): l:lrolh~r and ,m.er m~ uf Lh<• "''mo 59.("): Pr~mchantl wa' o noveliSt wilile lhe
rank. others a.te Um mat.1 gts.
38. fh):l! '" th~ only p·uu~ thai. " m 60. (<!):!tis tl1e only matrnal thai is oblaia.od
Rlph~betico 1 od~r and lhe reot' "tart from tr~es_
--
IType rvAI
8. Anwnborwhichwkndi,•;r':<,dhy-1,~, l(i
l. Rov1 ha• an annua l mmme of Rs. 2500. -3. lf that nnmhe r is
He opcnds 1~% on educat inn. 20% of lhe leave, a remain der
r<?rrm ning income 18 spent un housm g. dJVioible by 7 fmd th~ "" lf'hN ?
(a) 1-!1 (b) '17
Tht· rerntlir ur.g I fi% io dep""i ted in saving
(c) 147 (d) Cl'j
"chem~• and dw resl incom e i; spend on
purchas~d a b·k~ [UI Jh.SOO
food and cloths. How much perc~ntnge of 9. Mohan
incom~ do~B ~le sp<•ml on foo<'.. and clnth? mch1d ing.ea lc• lax n' :',IJ%. l'ind tlw
(a) &'5% (b) 61.2% oelling prit>O uf th~ bll<~
(o) W'/o (d) 55% (a) ]"(,_ 666.60 (b) R~. 600
2. A car goe6 35]<m in l hour, ne~t 270 kru (c) Rs. 1000 (d) fl,_ 900
~' 3 hro. ~ml next 80 km in 2\i hrs. Find 10. 1\Iy father di&trihut-ed R,, 280 in such n
the av~rage spc~d of the car? way that ~nch gll'l rocet'-'ed !\.>. ZO Rnd

om
rj;.) 5D. 23 kmJh. (b) 6l . .'i krnlh each boy Ro;. 10. If the lll"~~cr of boys is
(c) HO kmlh (d) Konen ftheab ovc JeBS thnn lhat of gill' oy ~- f1~d the
3. Mohan _s young er than his father by numb~r of boys?
~0 ym Fi yeao·o ago his father v.-a' 3 times (a) R (b) l-'i
'han him. Find t.he "-f:C ofhio futher al (c) 10 (di 7
presen t.' 11. A "tuden t wns ask<-d 1..0 Rdd Hi and

.c
(a) 30 yrs (b) 25 yrs. subtra ct 10 from a nnmbe l'. l-Ie by
(c) 3G yro (d) Nonco ftheah m·e miBtak e subtra cted I {l 'lml added 10 and
4. A train rno.• lor 2 hrs at t.he speed of found the an8wo r 14. What is tl><' righ\
"l che spe~d

ce
:-!5 kmlh. r1 tum for~)-, hro
nf fill kmlh and then runo :or 2'-', hm, at (b) 26
(a) ill
the 'peed of 70 km/h. l'i11d the averag e (c) 30 (d) .'12
gpeed o[ Lbe train'! 12. A studen \ attcmpt~d I 08<jU~o\1ciL, ill an
(a) 50 kmlh (b) 55 km/h
examin ation. In th" examm " l.ion €VUr:"
ra
(c) SO ktn.lh (d) .'56 ~7 kmlh_
wr<mg answe r we.' on·en 1 ~~ m inm mark
5. Toffee •.ue dis:nh uled among A, B. C, D omlrigh1 answe r we\; f'iven 1 mark. lflhe
srtd E ln sud·_ a "-""Y that A gets one lcs•
than B. C gets 5 more ch~n D and E gets stuclen l "cored e~ro marks lww many
wrong qn€st.ions were done by h.im?
m
3 more than n. If Ban<'. D's share are
(a) &:• (b) 81
equal who gut the maxim um numho r of
(c) 1'1\l (di Koncn<'th~sc
I o!JeB> ?
(a) 1\ (b) B 13. lf the area uf a given >qual'€ AHCD ;_, 3
xa

MD (d)C find the wtal area of the en tiro figure"


6, lfthe numbe r oft,wo di)(its arc revers ed;,
h~c<l!ll86 18 greate r 'han the mwthe r.
Fmd the numb~,·tfthe wm of the digit;
is equal to 4?
.e

(a) ~1 (b) 13
(c) 22 (d) 40
7. Rajasn iC.to Kabir, "If you gi,•e me T\s 2. J
w

oha 1l h:, double to you ~nd you will b£"Come


(a) 45'
tr1pple ro .\.tsha Hnw mucll muney doe"
0 (c) -i8
A1shahav~
U. ,\ >pider ohmils W mdl'eO ui a pol€ m 20
w

{ct) R• "i (h) Rs. 8


(a) H.o. 3 minut e, a<ld "lip., cluw11 2 metr.ce at th
(c) ll< 2
w
"''"Y mnm~nl. !fit tHkeo .~ hu todlP \L un (a) 29 (b) 13
J!o lop hnd the len~Lh ofth~ pole? (cJ 1H ldl 211
(il) 71 m~t.r·os (b) 72 ~1etree 22. F.nd thH value "f }; m th 10110" in;
(r') Rll nl~h·es (d) 90 m~ll·eo fig-ure.
A elass stal'l en nl ' .00 p.m ~nd la.;tcd till
3.52 p,\ll. In this durat 1nn ~ regula~ ~6 ~------16
perwrl~ are hel<l and 4 minut~s W~r<· alro
given Logo f1·mu one dabo t<J anoth er In 1' j_/ 3 3(;----._::.T
atten d the ccass What is t-h~

om
ex"n (a) ~:; ~b) 06
durat ion of e~ch peno d?
(c) 49 (d) ~2
(<,) ·11 ru {h) On: 2.~. !I gat<len has"" mony lluwcr hcmm g tr<>rs
(c_l (i~ m (d) 40 m
If 15 ap_::>Ce~ and 20 nTgane~ cost as much '" h·ull Lea ring Erbe'- 1rJ IL'"'" m·o old "nrJ
as 20 ap~]e, "nd L) orang es w )J ich of the % arc ~r"ft<'rl. Which ot' the followin~
following conci uhlon s is corn• r·t? interl erenc a' ar<' rl~fin11 dy true'!

.c
(a) Oran ge and apple have ideM ical (a) All II ower be_ ann r, '"""" are gmfl_ ,,a
pnces (b) Onh fruit beann~ tree~ m·~ ~rafted
rN Omng<>'s price is doubl e 'hat of apple (c) At leaol nnc half nf the rluw' '' be3lln g
(c) :'-lo conclu>ion can be draw n t.rceo ar~ oi1
ce
(d) ;\pple Js ch~aper thsn orang e (d) Alloflh~"e
24. On a ,jx puinl. 'cale if o 'lllde nl g<•tgrad<>
"· Ani! sold a cornmnd1ty .in Ro. 4GO ot the
loa" of 10'% .'\t whar ptir:<> rlW. he puroh a•c l! in Engli sh. groclc 0 Jl_ ]\,Tutm, ~t·,d·J B
in .Soie<1ee gr~de C in Suci"l Cii-ieme ,,nd
ra
!a) fu_ 495 (b) fu,, 500 gmdo fl in PT P:nd uut hio O'el all b'l'ade_
(d Rs. 105 (d) !'ione "fthe se !a) A (b) 0
A wagon has [J,e capac lty of 12 adull s or !c)C (d!R
20 ch!ldren_ How many adult s can be Ia a gmup of slud~nt," BI!IJ ofthe ln pel"OCd
m

board ed with i"J childr<Jn '! mall five oubjech1 21)1) faiierl in allth P
(a) .~ adullo (b) -5 adult ' "'hje cts 100 ill Engli sh only nne[ [50 in
(c) 6adu lt' (d) Nonn ofthe "c g,,~nce or.ly. Find the perc:~ntug-e
uf
If ihe follow ing '~ne> of numb ers is re"u lt,o nfLhe sch<,ol.
xa

\>rit: cn in th~ rever se ord<•r wh1ch (a) 51 1% tt) ';1%


nutnb~r "ill Oe l.hc oe,.,n l.h to the righ. (c) 00'1\ (,-{) None of liK•e
of
the fourt h numb er frum the left 26.. I11 a class. Ravi' s r.,~k i> 15ch froD Lh,,
I, 8, .), 9, 7. 4 10. 6, 2, ll. 13, 5, 11, 15 to~ and 2loc from che hott0m . !low many
.e

(a) -0 (b) 9 'tuden t.s are lher~ 1n tl>e d;~ss?


(c) ]';) (d) '2 (a) 31 (b) .16
In" row of child ren Pcrve el! is 7th form (c) .35 (d) .:;on<• Dftllese
the lef[., no hloo ;. fourth from the right. 27. The ~Miu of boy" a!ld gll'ls it1 a schrml i•
w

Whe_n oacl'_ uf thenl cx~hangc< their 1 : 8_ if there ~r~ ·!8ll boyo Jll I he scl:wol,
pooit.iun• Pcrye en will be 1-'ith from the find the numb er of ~iris ;
left. l'ind the t<Jtal numb er of c},,lrlre"
(a) 360
w

(a) 18 girls (11) J20


(11) '20 girls (r) 315 (1) Nm"' of Lh~'C
(c) 21gir l, (d) !!!gir ls 2B. A oar needs 12 litre o!' petn1l to c<JV<'r a
Sl!pp ly the rni»in ll: figtl''C mll1~ matri x?
distan ce uf u;~ km ;_ How much J1<'crul is
w

neede d to cover " distan ce of201 km.,,


' ; (a) l-J~htr~ (b) JB]Jt l"
" , (c) I~ litre (d) 11 lilrt'
z~. A contractor 1ln<l''l'1 """ Eo musn a worK 1~ u< · ~" eu cu '"~' • , ' '., '" -"" '"" "'" >'
f,~- Jl~ e>n~loyed HO mr>n l'or Lhi•./lftcr 02 n"- triA.
da10 he lu'lod r.naT 2i.~ o! the "urk h" (") H:n:, In) 12%
been ,·ompleotod_ !low ma11y work~r~ (c) 9}; (rl! ~nm uftheoe
should bo r~d•.Eced L<J fuu"h 'he wm·k just S9. l'lhal will be the 'l'''''d of the watn 11'"
_n l1We? !mal going at 9 kmih r 1" ol"ll waln e,-,,j : 2
00
(<>) (b) ~() kmolhr ill down•lreHrn "nd com~.> b~ck in
ld 2H (d) 36 tot~] thrr~ hour-·
30. WJ-,"1 pn<·encuge of 180 .'iO ;, :IIi I ' (a! 1 kmn, (bj 0 kmih
!o) ~0% (/J)2~%
(c) l .'i kmih (d) ;', k'llih
(r) 22-~0~, (d) 1\@~ofth~oc A :nan ""'"" 2<l% of Ius salat·y_ II due lu
.\ numbc·r E-3 o< mw:h gr-oacer than 17 as pnc~ """ h<• i nere~oe his nontl,- expen"~'
11" k:·e Lhau 57. Fmrl tl1P numher
u,. 2~% and he;, ;,ble lo ;aw only R, 20

(a) ~6 (b) 37 per month. Fmd hi' monthly onlnry_


(a) I!.,_ 400 (b) Rs. fiOO

om
lr) 40 (d) 1•!
(i) l(,_ 000 ~·I) It" ~~>~
Moh;m·, '"laJ'Y 1s JJ'l-u abo-cc llai"- 'fher1
'fh<" volume of a WRII 1' llil~~ cube
how much P'""enL a~~ Raja', balary ;, lcs•
melre.lts heighr. io 0 t1meo lui,o breadth
lhar_ }'iohan
anJ l~nglh is 7 t1m~o to ito h~1ght. flnd
-'") 20~, rN 2~ t'le breadth'
.'c) 21 (1i<3%) (rl) Noneol'chc;e
(a) 1 ill (OJ 4.5 ln
;\ rrduMion ul' 20% in the pJ'lc~ of r.JlplP'

.c
(<') J.6 m (d) Kone ofth~sc
enables a huycr w g<>l O<tH dozell more fm
.-'. ve_,,,e~ contains 100 lltr<;.-; of milk 50% of
lh_ fi('l Fi.nd th~ reduced pri•_oc per· dM~m of
il. 1.• Laken octt cwry d"v "JLd ~qual
nppk<? ,m,unl uf" aler lS ad dod to_ llu» nuoh

ce
(uJ Ro. 8 (b) n"- U ql.lantit_y of mJ..!ll. wi II r~mnin aftel' 3 days?
(c) Ro. 10 (J) Nm1~ofthese (a) 12 hire (b,l I fllitru
A d<--aler ould a Jnixu- fw R'- ,)40 Jo,,,ing (o) 12'l; l!lt~ (1) !1\', htrt
1()'•o_ At "-hat pric~ should he h ""e <old -o 4H_ On~ f1fcll of a number t'X<'t•Hh its -one
e01·n" 10+., proU'
ra
>Fventh by D•l. i"ind t.h~ nuJuhe,-
(a) lls ~fi[J (b) R" {i,-;o (<z) 2695 (b) ~lillli
(r) R,_ ~(l(l (d) C>lcne ofthcoc (r) 271Xl (d) ;<(J)
The differt•:>ce in selling price of a ro~io al If 20/X = X/15 thnn X= ?
gamo of 10% und I h% ;, n,_ 30. Fmd 'he
m
(a) 25 (b) ~·
price of lhe radiO? (e) 4!'> (r!) :m
(a) fll10 (b) 670 A msn oold 10 eggs fno· one rupee and thn<
(c) IY'O (d) 6l!O gailled 20% proflt_ Tlow many £ggo rlid he
xa

A >Um of mOlWY hecom~' "i.'6 of Jtaelt m 8 buyoforRc. l?


Y""'"' at cet•tam rate ot inl<'r<"'l
Fi ttd the (a) 12 (b) 11
race. (c) liJ (d) 16
(c,) ,-;'\;, (/J) 7%% A nnd D mv~st~d R.,_ ~000 and Rs. ~UOO
(c) S% (d) 12% r~'pcct.i\•ely in a pa;'tnership ],u,iness m
.e

37. The dilfercr.cc hNween sit:l~le which ,fl."'"'


Hleep:ng par<ner AL tloe ~n<l
compound. rate of L>Cero't nn a eerhin o:f one month hm.h rec~i,·e,l Re. I :'ill mch a>
sum ot money fur :< y~an at r;% rote of protlt Find ll"b ren.m~rntion fm· hte
intere<t '" n,_ ~5 Fin<! the sum_ work''
w

(") R". lGOeO (b) Rs 1:<111111 (a; Rs. 60 (b) no_ :Jo
lr; Rs. 10000 Iii) Rs. l!iOO (,•) Rs. 6ll (d) l\10n~ ot'the"P
38. Au al'lide i3 llored Rs. 1W 'l''lth n rli"-C>"Jlllll In an examination 40% 'tu<lenl.• f~il ill
w

uf 20%. Vi-1o" I "del iLi<~nal <l!bc0UU >honlcl l'l'!acho, 3Q')(, m J<;niish and 15% in !J<JIJL
w
Find the pa"" percentage o 4 appl~• "nd .~ omngco oost 37 paisa. Find
(a) 50% (b) 65% the cost of an orange?
(c) 8(1')!, (d) 45% (a) 3.V paisa (b) ~paisa
4S. The sumofageofa man and his"on i" 100 (c) 6pai"a (d) 7pa!sa
yr•. 30 yrs ago tho man was thr~e tim~" 50. Awrag~ age of 21 at.lld~nts ;, 15. If
a' •lld "" h" "m. Find th" ~g<> nfh1s 'on tcochcr·o age is inclwled the average age
at pre"""'? 1nnl"U<r"' b,.- l. J•'md tl:1e age of the
(a) 35 (b) 40 teacher'

om
(c) 50 (d) Noneofthe.•e (a) 40 Y"' (b) •13 yro
49. 11"3 apples and 4 oranges cos! 40 pahoa and (c) 21yro (d) 1Hyrs.

I EXPLANATORY ANS\'VERS I
l. (h): 1 0 nf 2fll10 = 2r>ll, Re. ~'.itln- :;c,u- ~~,:;u 6.(b): a+y=4 ,(l)
JOx2250 a+ 1f]v + 18- 10" + .v

.c
2U"'O uf ~250 ~ -- = R•. 4.';0 - 9a + 9y=- IS .. (2!
wo 9a+9y=-36
Rs. 2250- 150 = Rs. 1800
15xl800 ce iJ':'l l ;, mult.:phed b)~]
15% or 1800 = ----;;)i)-- = Rs. 270 lHy-;>4
_}' = 3
Rs. 1800- 270 = Ro. l5:l0
c=·i-3=1
Percent~ge <lf expenditUl'e on food and
"-!un,bor = 13
1530xl00
7. (c): SllWo>e K~bir ha" R., X
ra
cloth= - = ()1.2%
2500 2(X-2)=(X+~)
2. (a): 'l'otal clJotancccow:red 2X-1=XT2
=351270180 =~85km
X= 6
Tota.llimecaken =J+:I+2\'ihr8.
m

JI:Jof6i"=2
= 13/2 hrs. 8. (c): LCM of 4. 12, 1 c
J~5x2
Average Speed = D/T = - - - = 41:1 + 3 =51 not div1s1hl~ by 7
1.1 48 X 2 + 3 = gg not d:v"iblc by 7
xa

= h!l.2~ bnJhr. 48 X j + 3 = l-17 d"~s1blc by 7


3. (c): X-Y=20 ... I l) ~IJ0Xlll{l 200(1
9. (a): ~-~ = ---:; - R". 666.60
(X-~J=3(Y-"J)
X-.'i=~Y-1fi 10. (a): Tiny> =X. Gid< =X+ 2
.e

x.:w=-1~+~ !~) IO.e < 20 (x + ~J= ~NJ


1\o,., X- Y = 20 10x + 20x + ,10 = 280
X-:i'Y=-10 ~Ox- 24(l
= :m
~Y Y= I~ 8 x=
w

X=I0+20=:J5 11. (b): X-16110=H


4. (d): Totaldistance=2 x .% = 70 km. x~ ~!

=7/2XH0=210 X+l6-l0=26
w

=5/2x70=t•5km 12, (b): Sllppo<~ hio "mng ans"~rs wece X


Total distance= 155 km. Numhcr' of11ght answerg = lO~- X
Total trme = 2 + 7/2 + 51~ = H hra. (108-X).Jl3X~o
w

S']:i€00 = ·15518 = 56.87 kmt:=1 -1/3X=-108


5.(d): A=l.B=l·•I=2,D=2,C=5+2 X =o wg X :JI-1
= 7, X= ~1
1"=2+3=5 13. (b): Coum the number uf •quare' m the
!'Jgure all(\ ~mlli1Jl:, it by 3_ 6. ,,, 1, 3, ~. 1 are ntunenc al ·.•a.\u~'
Subje-ct c: Gr"d" :c-!urr.~r;e<l.C •;alur•
14. (a): In 20 minoJt.c" it climb< 8 meL-e.
Sxl80 Engli<+. E
In 180 m1nuteH ic climbs" ' "O Math< 0
= n metres Scien"" n
It olp.-; du"n 2 meLres in <•very 20 P.T. n
IC~onuteA >;, il> last r"ach :o th~ wp of &cia.\ SciPnc'e c
the po'e oxch.dee Fh1s prob]Pm_ So t.ulal Over an gr"de 181-5
length nt the pole= 7~ + t = T•; m ~~co 4 = 1! g1•ade
15. (d): 0.52- 1.00 = 2 52 i< t"t~l duration . 25. (a): Tnwl >cudent,<
Adjui<t~lent llme= 4 x -3 mi <\Utes= 12m = liOO + 200 + hJO + J 50 = 10~0
2 G~- 0 I~ = i.10 hnur> lOOx600
%age= - = 57.1%

om
120 + 40 o- 160 "''nutes 1050
IJumlwn nf ,,,,.h period " 160/1 = 40 26, (c): 20 I 15 = 35
minutes 27.(a) :4:3::4 80.'
16. (a): 10 "ppl~s + 20 Or,nge =20 "I' Pie>+ l•J 480x3
o=r
200-l: 'i0=20 ,\-15A
4/~
.
28, (b):(12 X 201)115 3"' 16!itre
'
= 1ROIX = X = - - = 3HO

.c
~ 0 = GA So orange= Apple
29. (c): 62- 32 = 30 doy•
lUOx450 1- 2.'3 - 113 work
17.(h): 90 =500
21:; work i< done in :)2 dayg by 60
20- 15 =~children

ce
i& (a):
workers
20 C~\ldr~n= !? ~~ult~ l work is done ill 32 dap Ly
5 childr~n oo :1 adult.<
HI. (b):'l'he nun-.hcr is being revers~d. It
(,(lxJxJ2
means th~ right side would be '
ra
cnn.;;u\ered lett. . ·. 113 work is don" in 30 days by
20. (a): Babloo Pet'\'een (60 X 3 X :J2)1(2 X 3 X 30) = ,)2 worke"
right- 4r.J, lth -left Reducti on= 60-32 = 28 workers
J610X LOO
Exchang<> .-LL - •lth =- ,;
m
30. (a): (:1ii_ lOx l 00)1180.-50= - ~o·e,
15th + •lth P'"ition c·f l:l"bloo 18050
17 +57
=l5+~1=l~girls 31. (h): - - = 74/2 = 37
21. (c): Fm,-. mw = l + (8 x 2) = 2()
'
xa

Second rnw = 9 + (J X ~) = ] 5 32, (n): tOO+ 25 = 125


Timdmw =- (j ·t (6 x 2)= 18 25xl00
%age= = 20%
22_ (a): Numh~rs arc giY~l\ on the lr·~T side and 125
their squares on cbc- righc sidr. 33, (<:): 20% of Ih 30 = fu. 10
He gets one dozen in H.s. 10
.e

~3. (~):Old trees al'<' ~14


112 oftl,e f)owc'r bearing trees= !14 ~ 540>< 100
34.(a): CI'befol '<'loas= -(iQO

"'
:-!14 - J 14 --' 1..-~ rem "i" left 1 0')', of 600 = 60 ""
w

Fruic b~m"l n~ ll'ees "re 1/2 of ch~ luCal. SP = 600 + 60 = 660


If ,,\l of thc•m ru·e old ilmear,s 112--' 112 35, (ll): 1U -to= D% = 30 then 100%
"'"''\d_ JOOxJo
w

l/4 _,_ l/2 = 814 ",., lJ!d m '"'"! · - - =WO


24.(d):O -~ 11 C: DEw u oix poim sc"l" 36. (a): II~><' I 00 = 140- ]00 = Rs. 40 inn-,aoe
w
=h% lG12S 2-0~X'. x- =
37. (<o): Compounr1 Tnt~l'~ol on R.,._ tOO X'' = 6.;
X= 4
= 111Ux2lx21 -l(}O
Jl_ (d): 100- 00 =50= Fir"t d~y
2Ux20 ii~- 25 = 1!1 =second day
= 4~1(1- wo = 41/1 ~'J- 1~\i = 12!-<; ~ Thi:rd da:>
Simple inter<:"' on Rs. J 00 4~. (")'JIG - 1/"io= 1&1

om
=0x2X l00/1Q (I=l0
1/:J.'i ~ Hi-1
IJiffet"n"" = ·11/-1 - 10 - 1/4 154 x3:<
1-'·1 = 25 ;o- -

~H. (n):
100 = 2Sx -1" IOO=lh . 1()1100
20% of 1GO -= :;o
150 -80=R~.120
4+. (d): X' ~
'
= ~7" 35 = 26~5
---
20 x ·!5. X = ~·(9l'O) co 80

.c
UO- 108 = R~- 12 as di,·rount IOOxlOO
DJ>ooun ·~age= 12 x 100/120 = 10%
4,;. (a): C.P. of I~ rcc'= Qi-,-
~9. (<:): Let lho spo~d of the wore~= X co 2~0:'~ p<.ioe
SpE>eJ oftho boat dnwn stream- 9 +X
Speed Q[ lho boat upetroom = 0 - X
D1stanru cn•;erecl ~ 12 km
ce c_p_ of 1 egg= 2.)Uf3 x J.IIO = ~0/3 P"'"e

How mnny in 100 P""" = 2o = 12


IGOx3

T\me = DI.0 tonce Covcr~O/speed -16. (a): [{ntio of A a no: B :.L2 or HO% antl40'.\ ,
profit.
ra
12
12 Let th~ wofn. o;· A- .~X Rnrl B- 2X
:J/1 =--
9!-X
·-- 9-X B lo ]~"·" I han 1\ by X m catio
[3(8+X)-<~-X)=lllf. 12X+lO A 3X +2X +X= :JOO_ SoX= ~0
-+ 12X]I (~I+ X) (9- X) 47. (d): •10%- 151·, = 15% + 30% = .')~%
m

3(9+X)(9-X)~2H:l
100- .55= 45%
(~-+ X)i9-X )-72 48.(b): a~I>=JOO .11)
81 X3=72 n-30-~(o-30)
xa

X2=-S1 + 72 a-:JII- 3b-90


X'=9.X =3 a-.3b= -60
40. (a): &penscs hej:ore pri<"' rise= lDIJ- 25 = 75 a+U=IO O
25% inm·ea&e in expcnde " = 100 -< 25 -1U HlO
=Rs.12~. b- 40
.e

75xl2~ 375 49, (d): :l Appks + 1 omngcs = 40 p:>is·c


100 4 4 Aprle + :; urung-e = :17 paise
·oa+4h =40
w

S:,vmg = 100-375 = 25
·1~ + :Jb = J7
If &<,-ing io 2511 lh~n salarv" 100
If ,,wing is 2'\ tl1~n '~lary --
' ' 11a + 1r.b = :on
12o+~1=11l
w

100xl5x 4 Th = ·19
h = 7 IJai.oe
25 50. (a): Tolal ago ~fl>up-- IF, x 2•1 = :J(iO Y!"O.
w

= Ro. 400 Total "go of 10,1'.' tndL,din g te,,·hcr


41. (a): Let the h1·~:Hlth be X
=1flx~5
HeighL = GX
= 100
Ltmgth = 7 x liX = 12 Teacher 'eage =-400- 360 '= 40 years_
]Type rvs]
12 (0) n
l.lf84 Kl3=8 ,31x1 B-G,2 8xl1= R. (a)
~ > (c) cl3 (,!) 20
then ~r, x 22
(a) 86 (b) SC: 11. If~ ruoans '~""'~ l.he hrst nnmbcr and
(c) 7 {<I) ll then Ill u ltiply 1! hy th~ next nu 01 ber and
2- ffl=3.2=5.3=7.·1=~.lhu.7 ° CJ mcam '1mlhpl y th,. pr·od\1~~- with th<•

(a) 1;; ('J) 13 second mu~be1· and oublmc t the seccod


(c) 17 (d) 11 nl.Omher from Lhe prDd·~ct nf the h''"
.1. 1!'1~1 2=4,3 = 10and 4=22, thon numbOl'" then find the value of 2 /'. :1 Cl G'!
5 = 'I (a; 5~ (bJ 6(J
(b) M (c) 7 (d) 24
(a) 39

om
(c·) +4 (d) 1~ 12- On Lh•' h""" of the quc.;tw n ll fmd the
1. lnoert the arithm~CJCol si~n" m Lne \·olue of.'J DJD 2 il2 '!
((]) --1!!1 ib) 4-41
f"llow1 r_ g n tUM neal J'J gu:rc .
(c:; ClfiS (d) Kane ol'thcse
9G3= 27
(a) K (b) --, + 13. 1'h" rallo of boys ,m<l ~ll'l< m ~ och~ol i"
(,') (d)-.- <1 2_ ZO% nf hoys 'md !J"'o of ~ll'l' "'"

.c
5. ln"ert the num~ricsl signs m che schobrs h1p hul<lcrs The p~rcenwgc ol'
lollowHlg numel'lc~l figur~- H l! ~ I = 14 student s who '"" Bcholarohip holJero nr" '
(a)-,- .- {b)-,- .x (a! 4~ (6) JS
lrl 60 (d) 22

ce
(c) x,-,- (d)+,- ,x
ll, IJ'4x2- !=B,'?Y 8=·4, lY:J=: , thenfln d 11. A bag LXlllltlJM ~n ~qu"l mW1\l(r d cno
the volu~ of 7 " 21 '1 Tupee, 50 pais~ ond 2c, pa1•aro Jns. !!'the
(a) 42 (b) 11 valu€ ofm()JlC> m the bag'" R&_ 35.l'wd
(c) 3 (d) r,8 the t.,;tnl number of C">ino of each r.n,e?
ra
7. If+ means X. - mcmL> ->, -mean s ~!ld (a) 7 (l,J 40
(c) .'10 (d) 10
xmcan s- fi,-,dlh evahl< 'Of'i+G -Zx 12
~ 1. 15. If A- B w.con8 A JO the daught er of ll.
Ax B me~110 ,-'. 1s \he ~on of 11 and A- L>
m
(II) 23 rntan~ A"' tl>< wif~ nf B then P Y Q --!:!
mean"' !
(a) fl is th·-' fatr.N of P
xa

(1,) Q" the father of P


(c) .'\._ooo nofQ
Dircct iuns for questio ns 8 and 9 :
(d) Non~c.fclw<c
If> >lands for= < ~oonds foo· ;<
16. Char_gP tb~ oJgn to f1nd tlt€ cquatw n 2/i -
X swnd' for> +"land o !01' <
(:J-c4 )+(2xZ j=0
.e

= ot.ond• for f - ~c~ncls fm -1;


(a) CJ.cmgc + mto K(b) Ch~ng€ x into-
8. l'u1d the \'ahH' uf a x ~ ;> Y
(1') Chang e- ;nto < (d) Changr + l!lto
(a) a-B= Y (OJ ll>~>Y
17. If+ !~can, d"~de. " mtans m1nuo, ~
w

(c) cr-fl> Y (d,l Noneu fthe•e


means multJpl y and- meno plub then
9. o>bXcm~a:~s
(h)a> b><· find the ,-a[u~ nf 9 + 3- 'l- 8 x 1
('!)a-l ;bl(c
(u) 15 (b) 17
w

(c) a=h> c (d) ,\lloft.h ese


1G, ~X 2 = 20. F1nd (c) 1~'1 (d) W
10. jf:J X 6 = 18, 5 X~=
cbe valw; o0 4 x ~ = '1 18. WhJCh uf th~ two "1~11e "hould he chan~~d
w
Lo make the ettu"tion correct. m"n can dig it m i) dHys an~ third mon
((; 3)+(1-2+1 3)-i-(7>< 2)=21 can dig it in 0 days woektng alone How
(a)"- (b)"~ much t.ime will 'hey tak~ "' rl1c- "
(c) + (d) x- toget.iler ?
19. If q means>. D means< and i\ means- (a) !OI?clayo (b) :;Oi1-3days
and 1f A c:::J B Bq(' and D d A, then which (c) IQ/~Oda}o (cJ) ;'>!un~oftheoe
oft he following is correct? 23. What sign 5houlcl be changed. lo make
(a) Bc.D (bJ ll=.ll. the equation 5 + 6"' J- 12 x 2 = 17.

om
(c) C > ll (d) No no oftheae oorrecl'!
!Iow many pillars are n~edod to cost rue' a (a) + ~ 0)
bnd:gc of 300 m"tre long. if pillars are eil (c) + x (d) Non~ofthese
a ilistance oi 12% rr.etreb each 24. lt5x~=~8.3x'i= 12,8XG7~51C>en
(a) 22 (b) 24 ftnd the valw• nf 1~ x J:J?
(c) 2ii (d) I\one ofthcso (a) 169 (b) J.)O

.c
If 12 = lll and :12 = 26, Lho~ 22 = '! (c) 140 !r!) 144
(a) 20 (I>) Hi 25. Jf .J6 X 11 ~ fl. 37 X 13 = ~- 4~ X l? = ~
(e) 15 (d) 17 lhBn hnd ch~ \'Ulne of ~7 x 77-?
ce
22. A man digs" trench in -~ clays, another (a) 1 (/J) -3
(c) 4 (di -'i
IEXPLA l\ATOR Y ANSWE RS!
ra
l.(c)' (8+4)-(I+ 3J=8.(3+ 7) -(I+:l)= S.(d): <>~Jl>y
6and.(5+ 6)- (2 +2) ~ 7
a>~=yltm~"""
2. (a): Assigned. cod.~s are incr~asing at o.n u •• ~ > y
int<?nla] ol' ~- 9.(a):a>b ><c
m

3. (d): I, 2, 3, 4, r,, (Digit') a=iJ>r


1, 1, IO. 22 46 (CodP.), Gap is hcmg a>b"c
doubled vt ev~r_,. diglt. 10, (d): {:J + 6) X 2"' 1~.
xa

1. (a): Always apply th~ rule• of BOD :MAS in (5 •:JJX2=Hi .


euch typ~ ~f problems. In this problem (8+2)X2=~0
mulo.iplicatlOU will h<- cion~ Grot then II. (a): 2,~ 3 Cl ~
comeo the operation u[ adrl1o.ion and 2X2X-~c:::J5
llnally subtradion , !.e .. 9 + 6 x 3 = U
.e

12 D D
5. (d): The explanatwn io in answer No.1, =[~X0-5=.')6
l.e.,8+S -2xJ=l4 l2.(c): fic:::JJD 2 t. 2"' (,'j X -3- :J D2d 2
6. (c): The first figur<e 1a dh·i"'r and the "([742X2 -2).!.2
w

""""nd one is diY!dend =(~2X 22) X2=,181 x2=96S


21-i-4 = 6, 8 -c2 =4. -3 ~I =:J 13. (d): 3: 2of lOll= 60 anJ 40. 20% of60 = l~
7.(a): 4+G-2xt2~4 and 2~}, of 10- J[·
w

X + ~
4x6+2~ 12-4
X X X 7X
-+,-+-= -%or
4XIi+1/6 -4 I 2 4 4
w

21+1/li-1 ":Hi or 7X
"3!Jx1uc X=20
145/6 .. 4 = 121/fi = 15. (a): P X Q = l' lo lhe ,,on ui'Q and Q i6 wif,•
of S. S io :he father of P.
So tbe numb~r wl11ch is h<>tween
Hl.(a ):2H -{3 +4) x (2 x Z) =0
(1)9+~ 4-~x2
21- H2 oho11lfl be 4 pomLless. '·'" . l2 = 1 R
17.(~):
(ii)9"~X3+R-2 I I 1 10+6 +5 21 or -da, IO
22. (a): -;:~-::1"-=---=- ·;.
(i;j)9X1/3X~I·8-2 oo6 J(l Jlj -;·
(ir!) 9+H -2= 15 23. (d):)~ 6- 3 + 12 + ~' l7
18. (d): 24.(d ): (4-l) (R-1 )=28
19.(a ):A< B
ll>C
D = .\
'"
(~)
{3)
(J-1 }(7- 1)"1 2
(b-1 )(6- 1)=3 5
th"n (1.3 l) ('.3- n = 144
g,.c , B>A~n 25.(a ): (5+6 )-(1 +ll= 9
20. (c): 300- 2512 +one p11lat• = 300 x 2125 + (::+7)-(1+~)=6
one !Jillar = 2~ (4+2 ) ·(lt- 2)-·3
21. (c): Here 2 pomt d~creases at. everv <-~""· lhen(8+7)-(~<7)~1

om
e.g.• 12 = l 0 oml 32 = 26.

.c
ce
ra
m
xa
.e
w
w
w

You might also like